Pulmology Practice Q's (Exam 1)

Pataasin ang iyong marka sa homework at exams ngayon gamit ang Quizwiz!

______________________ is the most frequent cause of "typical" bacterial pneumonia in children of all ages.

Streptococcus pneumoniae

CURB-65 is an acronym used for community-acquired pneumonia that stands for confusion, uremia, elevated respiratory rate, low blood pressure, and age greater than 65 years. Patients with a score ≥______ may need hospitalization.

two

Atypical pneumonia is also known as _________________ because it generally follows a more indolent course.

"walking" pneumonia

Your patient presents to the ER after accidental ingestion of a marble which is causing severe respiratory distress. Upon CXR, you are able to visualize a foreign body in the 3rd generation segmental airway. Should you use flexible or rigid bronchoscopy?

-Flexible bronchoscopy is indicated in this scenario. -Rigid bronchoscopy is only able to reach the trachea or proximal bronchus and cannot reach below the carina. -Flexible bronchoscopy is able to be performed at the bedside with/without sedation.

Serum pH > 7.44 is defined as ___________

Alkalemia *Emia is the state of the pH in the blood

Macrolides, doxycycline, or fluoroquinolones are used to treat __________ pneumonia.

Atypical

A random minor mutation in the surface protein genes in the influenza virus is called antigenic (shift/drift) ____________, and causes epidemics.

Drift

What percentage of FBA cases are in children? A. 40% B. 50% C. 70% D. 80%

D. 80%

_______________ is the drug of choice in mild-moderate blastomycosis.

Itraconazole

Aspiration of gastric contents causing chemical pneumonitis is referred to as ______________ syndrome.

Mendelson's syndrome

_____________ is a medical procedure in which the pleural space is artificially obliterated by causing the visceral & parietal pleura to stick together.

Pleurodesis You may need pleurodesis if you've had a recurring collapsed lung (pneumothorax) or an ongoing buildup of fluid around your lungs (pleural effusion).

In foreign body aspiration, only 10% of foreign bodies will present as (radiopaque/radiolucent) _______________.

Radiopaque (visible on CXR) Radiolucent (nuts, foot particles) ARE NOT detected with CXR

Surfactant (reduces/increases) ___________ surface tension which determines the patency of the lungs.

Reduces

Silhouette sign

This CXR exhibits a _______________ sign.

A low VQ ratio is when __________ exceeds ___________.

Perfusion exceeds ventilation (not breathing enough)

Patients with empyema not responding to simple chest-tube drainage and antibiotic therapy may need what treatment?

A "Clagett" procedure that consists of open-window thoracostomy, antibiotic irrigation and closure of the window.

A 46-year-old man comes to the emergency department after three days of difficulty breathing. He has a cough productive of yellow phlegm and his dyspnea worsens with minimal exertion. He has experienced no fever, chills or chest pain. He is otherwise generally well except from a morning cough that brings up white sputum which he has had for many years. He has a 40-pack-year smoking history and has not been vaccinated for pneumonia or influenza. On examination he has an oxygen saturation of 88% on room air, no cyanosis, a prolonged expiratory phase and auscultation elicits expiratory wheezes bilaterally. What is the most likely diagnosis? A. Acute exacerbation of chronic obstructive pulmonary disease B. Bronchial pneumonia C. Bronchiectasis D. Congestive heart failure E. Viral pneumonia

A. Acute exacerbation of chronic obstructive pulmonary disease Chronic obstructive pulmonary disease exacerbations present with dyspnea on exertion, productive cough, chest discomfort, wheezing, and hypoxia. It is most common in smokers and is the fourth leading cause of death in the United States.

A 45-year-old man is admitted to the hospital with shortness of breath and found to have a pleural effusion on chest X-ray. Thoracentesis is performed and the fluid is sent for laboratory testing. Results of the fluid analysis are: pleural protein 8.0, serum protein 6.5, pleural LDH 500, serum LDH 100. Based on these findings, which of the following is the most likely etiology of the pleural effusion? A. Bacterial B. Congestive heart failure (CHF) C. Cirrhosis D. Nephrotic syndrome E. Pulmonary embolism

A. Bacterial Pleural effusions are collections of fluid in the lungs. Exudative effusions are characterized by a fluid protein to serum ratio >0.5, fluid LDH to serum LDH ratio >0.6, and LDH >2/3 the normal serum upper limit.

Which sign is classically suggestive of acute pulmonary edema? A. Bat wing appearance B. Deep sulcus sign C. Costophrenic blunting D. Lobar consolidation

A. Bat wing appearance The bat-wing lung appearance is a sign of acute pulmonary edema.

According to the advisory committee on immunization practices (ACIP), which pneumococcal vaccines are recommended for an immunocompetent 65-year-old patient with cochlear implants and no other medical comorbidities? A. Both pneumococcal conjugate vaccine and pneumococcal polysaccharide vaccine B. Neither pneumococcal conjugate vaccine nor pneumococcal polysaccharide vaccine C. Pneumococcal conjugate vaccine alone D. Pneumococcal polysaccharide vaccine alone

A. Both pneumococcal conjugate vaccine and pneumococcal polysaccharide vaccine What atypical pathogens most commonly cause community-acquired pneumonia? Answer: Mycoplasma pneumoniae, Legionella pneumophila, and Chlamydia pneumoniae.

A 57-year-old female presents to the emergency department with a worsening cough, shortness of breath, and night sweats. In addition, the patient has experienced a 10 lb weight loss over the past 2 months despite having made no changes to her diet. She was recently released from incarceration and smokes one pack of cigarettes per day. Past medical history is notable for hypertension and hyperlipidemia. The patient was born in Mexico and had a positive PPD test as an adolescent. Chest x-ray is demonstrated below. Which of the following best describes the radiographic finding above? A. Calcified mycobacterial lesion and inspilateral calcified hilar lymph nodes B. Fibrocaseous cavitary lesion in the upper lobes C. Hilar lymphadenopathy and a primary focus of mycobacterial infection D. Cavitary caseating lesion E. Primary focus of mycobacterial infection

A. Calcified mycobacterial lesion and inspilateral calcified hilar lymph nodes Major takeaway There are three primary, progressive lesions noted in pulmonary tuberculosis : Ghon focus, Ghon complex, and Ranke Complex. Main explanation This patient has findings concerning for pulmonary tuberculosis including night sweats, fevers, and cough. The chest x-ray demonstrates a calcified lesion in the axillary region of the right lung field and ipsilateral calcified hilar lymph nodes, which is referred to as a Ranke complex. There are three progressive lesions noted in pulmonary tuberculosis: Ghon focus, Ghon complex, and Ranke Complex. A Ghon focus appears first and is characterized by a primary granulomatous infection filled with mycobacteria. As mycobacteria escape into nearby pulmonary lymph nodes, they enlarge and can become radiographically apparent. Together, a Ghon focus and hilar lymphadenopathy is referred to as a Ghon complex and is typical of primary pulmonary tuberculosis. Over time, the Ghon complex scars, becomes calcified, and is even more radiographically apparent. This pathologic variant is then referred to as a Ranke complex, which generally contains less mycobacteria than a Ghon complex but can still yield features of active pulmonary tuberculosis.

A 65-year-old woman is admitted to the hospital because of severe shortness of breath that suddenly began 1 hour ago. She also has left-sided chest pain that is worse on inspiration. Medical history includes dilated cardiomyopathy and congestive heart failure. She was also recently mechanically ventilated in the intensive care unit for 2 weeks after developing a severe infection following resection of an ischemic portion of small bowel. Her temperature is 36.8°C (98°F), pulse is 119/min, respirations are 27/min, and blood pressure is 91/78 mm Hg. Examination shows that the left side of her chest is hyper-resonant to percussion. Chest X-ray is obtained. Which of the following is most likely a risk factor for her current condition? A. Chronic mechanical ventilation B. Congestive heart failure C. Dilated cardiomyopathy D. Short stature E. Female gender

A. Chronic mechanical ventilation Tension pneumothorax is a serious potential complication of long-term mechanical ventilation. The condition requires prompt treatment with decompression catheter placement in the second rib interspace in the midclavicular line.

A 35-year-old woman develops dyspnea 4 days after thoracic surgery. Patient starts coughing with a yellowish sputum. Chest x-ray shows pleural effusion and contralateral mediastinal shift. Thoracentesis drains a milky white fluid with no odor and a triglyceride level of 115 mg/dL. Which of the following is the most likely cause of this patient's pleural effusion? A. Chylothorax B. Congestive heart failure C. Empyema D. Hemothorax E. Pneumonia

A. Chylothorax Chylothorax (or chyle leak) is a type of pleural effusion. It results from lymph formed in the digestive system (chyle) accumulating in the pleural cavity due to either disruption or obstruction of the thoracic duct. Chylothorax is related with malignancy and recent thoracic surgery.

A 55-year-old female presents to the emergency department with worsening cough and hemoptysis. The patient has had a cough for months, but over the past week, it has become more severe and produces bright-red blood. She is originally from Bolivia, where she worked as a nurse, and she immigrated to Ohio several months ago. Past medical history is unremarkable, and she does not smoke or use illicit drugs. Temperature is 38.3°C (101°F), pulse is 110/min, respirations are 22/min, blood pressure is 90/64 mmHg, and O2 saturation is 92% on room air. Physical exam demonstrates an ill-appearing, cachectic woman. Cardiopulmonary exam is notable for diffuse rales throughout the lung fields, and mental status exam demonstrates a confused woman who is unable to recall the year or month. Abdominal exam is remarkable for hepatosplenomegaly. What is the most likely diagnosis? A. Disseminated infection with acid fast aerobe B. Systemic infection with a Gram negative diplococci C. Metastatic renal cell carcinoma D. Systemic sarcoidosis E. Mycotic infection

A. Disseminated infection with acid fast aerobe Systemic miliary tuberculosis refers to disseminated tuberculosis infection and presents with signs of multiorgan dysfunction. Radiographically, the disease is characterized by reticulonodular infiltrates distributed uniformly throughout the lungs. This patient's history of working as a nurse in an tuberculosis endemic country, in conjunction with her chronic cough, hemoptysis, and signs of shock, raises concern for systemic miliary tuberculosis.

A 56-year-old Hispanic man comes to the office complaining of recurrent headaches, fever, and vision disturbances. His medical history is relevant to positive HIV. Physical exam shows a malnourished appearance, neck rigidity, and a positive Brudziński sign. A lumbar puncture is obtained and cultured. The microorganism grows in Sabouraud agar and has a microscopic appearance of an encapsulated yeast with narrow-based budding. Which of the following stains would most likely reveal the organism responsible for this patient's symptoms? A. India ink stain B. Periodic acid-Schiff stain C. Giemsa stain D. Ziehl-Neelsen stain E. Gram stain

A. India ink stain Cryptococcus neoformans is an encapsulated yeast with narrow-based budding, that is most commonly found in soil or pigeon droppings. The infection can be acquired through inhalation with hematogenous dissemination to meninges, that primarily affects immunocompromised patients. Treatment for Cryptococcus neoformans meningitis is usually with the combination of amphotericin B and flucytosine, that is often times followed by fluconazole. India ink stain is a microbiology laboratory test useful to confirm the morphology of Cryptococcus neoformans.

A 32-year-old male presents to his corporate health office prior to starting a new job as a perfusion technologist. As part of the onboarding process, he receives screening for tuberculosis using a Mantoux skin test (also termed purified protein derivative test). The patient inquires about how the Mantoux skin test results will be interpreted. Which of the following is most suggestive of a positive test? A. Induration of 15 mm in an otherwise healthy patient B. Induration of 5 mm in a patient who immigrated from Mexico ten years ago C. Induration of 10 mm in an otherwise healthy patient D. Induration of 1 mm in an HIV positive patient E. Induration of 5 mm in a patient with IV drug use

A. Induration of 15 mm in an otherwise healthy patient The Mantoux skin test, or purified protein derivative test, is a tuberculin skin test commonly used to assess for latent tuberculosis infection, particularly in healthcare workers and other high-risk individuals. Any individual with an induration ≥15 mm is considered to have a positive test result.

A 57-year-old male presents to the primary care physician for follow-up after recently being initiated on therapy for Mycobacterium tuberculosis infection. Since initiating treatment, the patient notes his sweat looks orange-red. Past medical history is notable for HIV and atrial fibrillation, for which he is taking HAART therapy and warfarin, respectively. Temperature is 37.0°C (98.6°F), pulse is 65/min, respirations are 14/min, blood pressure is 129/74 mmHg, and O2 saturation is 96% on room air. Physical exam is unremarkable. Which of the following best describes the mechanism of action of the medication causing these side effects? A. Inhibition of bacterial DNA-dependent RNA synthesis B. Inhibition of mycolic acid synthesis C. Conversion into pyrazinoic acid in susceptible mycobacterial strains D. Inhibition of arabinosyl transferase E. Inhibition of 30s ribosomal subunit

A. Inhibition of bacterial DNA-dependent RNA synthesis Major takeaway Rifampin inhibits bacterial DNA-dependent RNA synthesis. Its side effects include induction of cytochrome p450, hepatotoxicity, and red-orange bodily fluids. This patient with a history of HIV and atrial fibrillation treated with warfarin presents after initiation of RIPE therapy (rifampin, isoniazid, pyrazinamide, and ethambutol). He complains of red-orange tinged bodily fluids, and his prothrombin time is subtherapeutic despite anticoagulant use. These side effects are consistent with those of rifampin.

A 52-year-old man comes to the emergency department with his wife because he was found unresponsive at home in a pool of vomit and bloody sputum. His wife says that he drinks approximately a case of beer everyday. His temperature is 39.1 oC (102.4°F), pulse is 105 /min, respiratory rate is 25/min, and blood pressure is 111/81 mmHg. Physical examination shows significant difficulty breathing with a very foul smelling breath. A cardiac examination shows no murmurs with a normal S1 and S2. A chest X-ray is provided below. Which of the following is the most likely pathogen responsible for his condition? A. Klebsiella pneumoniae B. Chlamydophila pneumoniae C. Staphylococcus aureus D. Psuedomonas aeruginosa E. Mycoplasma pneumoniae

A. Klebsiella pneumoniae Chronic alcoholism is the most common risk factor forinfection by K. pneumoniae, because chronic alcoholics are at higher risk for aspiration and aspiration pneumonia than the general population. Patients with Klebsiella pneumonia classically develop lung abscesses and have a cough productive of bloody, "currant jelly" sputum.

Which of the pulmonary hila is usually more superior? A. Left B. Right

A. Left The left hilum is usually 1-2cm higher than the right

A 30-year-old woman comes to the emergency department because she attempted to commit suicide by swallowing a handful of aspirins. She has rapid breathing, vomiting, dehydration, fever, double vision, and is feeling faint. She is also complaining of ringing in her ears and loss of hearing. Which of the following acid-base disturbances will be most likely observed in this patient? A. Mixed respiratory alkalosis with metabolic acidosis B. Metabolic acidosis C. Respiratory alkalosis D. No acid-base abnormality E. Mixed respiratory acidosis with metabolic alkalosis

A. Mixed respiratory alkalosis with metabolic acidosis Aspirin causes a mixed acid-base disorder. It causes a metabolic acidosis by uncoupling oxidative phosphorylation, leading to a build up of organic acids in the blood. It also causes a respiratory alkalosis by stimulating the medullary respiratory centers, resulting in hyperventilation.

Which of the following describes the tuberculosis drugs that can be taken with OR without food? A. PZA and EMB B. RIF C. INH D. RIF and INH

A. PZA and EMB Rifampin (RIF) and (INH) should both be taken on an empty stomach as food reduces absorption

What does an abnormally sharp costophrenic angle (deep sulcus sign) suggest? A. Pneumothorax B. Consolidation C. Collapse D. Pleural effusion

A. Pneumothorax

A patient who is HIV+ has had a PPD skin test. You note a 7-mm area of induration at the site of the skin test. You should interpret the results as: A. Positive B. Negative C. Inconclusive D. The need for repeat testing

A. Positive The patient with HIV+ status is considered to have positive results on PPD skin test with an area greater than 5-mm of induration. The client with HIV is immunosuppressed, making a smaller area of induration positive for this type of client.

A 38-year-old man comes to the office because of a persistent headache that has worsened in the last three days. His temperature is 38.1°C (100.6°F), pulse is 75/min, respirations are 12/min, and blood pressure is 120/75 mm Hg. He requests that the lights be dimmed. Acetaminophen helps with the pain temporarily. He has also been sweating profusely at night and says his muscles ache. Rales are heard on auscultation. He is the owner of a pet store, where he began remodeling the aviary last week. He does not know of any sick contacts. Which of the following is the most likely diagnosis? A. Psittacosis B. Legionnaires' disease C. Migraine D. Q fever E. Chlamydia pneumoniae infection

A. Psittacosis Psittacosis is a disease caused by Chlamydia psittaci, an obligate intracellular bacterium transmitted primarily from birds. This diagnosis should be considered in a patient with fever, headache, dry cough, and systemic symptoms who has exposure to birds.

A 48-year-old woman comes to the clinic because of increasing breathlessness. She returned from a trip to Hong Kong, China 2 weeks ago and reports fevers, muscle aches, headache and a dry cough for the last week. Over the last 2 days she had noticed increasingly severe breathlessness. Past medical history is noncontributory. CXR shows bilateral patchy infiltrates. Laboratory studies show leukopenia. Which of the following is the most likely cause of this patient's symptoms? A. SARS-CoV B. Zika virus C. Legionella pneumophila D. Dengue virus E. Plasmodium species

A. SARS-CoV SARS is characterized by up to a week of high fever and non-specific viral symptoms followed by development of a dry cough and dyspnea. The incubation period is 2-10 days. CXR shows bilateral patchy infiltrates consistent with atypical pneumonia and laboratory studies show leukopenia.

In a patient with atelectasis, would the trachea deviate toward or away from the affected lung? A. Toward B. Away

A. Toward Significant atelectasis results in volume loss, resulting in deviation of the trachea toward the affected lung.

A 59 y/o F with COPD and diabetes presents to the ED with SOB. You want to evaluate her condition what method should be used? Pulse oximetry VBG ABG End tidal CO2

ABG Need to know pH, bicarb and PCO2, and oxygenation

Serum pH < 7.36 is defined as ___________

Acidemia *Emia is the state of the pH in the blood

____________ is a pathological process that lower bicarbonate and raises PaCO2

Acidosis *OSIS is the process that leads to an -emia

A 67-year-old man presents to the emergency department with acute onset of fever, chills, productive cough, and difficulty breathing. Physical examination reveals that the patient is hypotensive (BP 89/54 mm Hg) and tachycardic. He has a respiratory rate of 25 breaths/minute. Lung examination reveals inspiratory crackles with percussive dullness noted at the left lower lobe. Lab results show some mild leukocytosis with a blood urea nitrogen level of 22 mg/dL. Labs are otherwise normal. Radiograph of the chest shows a left lower lobar consolidation with air bronchograms. You diagnose community-acquired pneumonia. Which of the following is the most appropriate next step in management? Admit patient for chest physiotherapy Admit patient for intravenous antibiotic therapy Admit patient for observation Admit patient for oxygen therapy

Admit patient for intravenous antibiotic therapy CURB-65 uses five parameters to estimate a 30-day mortality score of patients diagnosed with community-acquired pneumonia: confusion, uremia ( > 19 mg/dL or > 7 mmol/L), respiratory rate ( ≥ 30), blood pressure (< 90 mm Hg systolic or ≤ 60 mm Hg diastolic), and age ≥ 65 years. Each of these parameters is assigned a score of 1 for a possible total of 5. A patient with a score of 0 or 1 (30-day mortality rate of 1.5%) is treated outpatient using a macrolide or doxycycline. A fluoroquinolone should be used for immunocompromised patients or patients recently treated with antibiotics. A patient with a score of 2 has a mortality rate of 9.2% and should be admitted for antibiotic therapy or for observation. A patient with a score of ≥ 3 has a mortality rate of 22% and should be admitted for IV antibiotic therapy with consideration for ICU admission for scores of 4 and 5. The patient in the scenario has a score of 3 and therefore needs to be admitted for IV antibiotic therapy using either a beta-lactam, ertapenem or a respiratory fluoroquinolone. Hospitalized patients with risk factors for drug resistance may require antipseudomonal beta-lactam or carbapenem plus a respiratory quinolone. Risk factors for methicillin-resistant staphylococcus aureus requires an addition of vancomycin or linezolid. Community-acquired Pneumonia (CAP) Patient will be complaining of sudden onset of cough, fatigue, and fever PE will show focal rales with auscultation of lung fields Most commonly caused by Streptococcus pneumoniae Confirm dx with CXR or ultrasound (operator-dependent) CURB-65 to stratify risk, inpatient vs. outpatient management Tx: 2019 IDSA guidelines for adult CAP, guidelines for empiric pediatric CAP

____________ is a pathological process that raises bicarbonate and lowers PaCO2

Alkalosis *OSIS is the process that leads to an -emia

Which of the following is an appropriate antibiotic for the treatment of community-acquired pneumonia in an outpatient setting, assuming the patient is not pregnant and has no comorbidities or recent antibiotic use? Amoxicillin Ampicillin Cephalexin Levofloxacin

Amoxicillin Community-acquired Pneumonia (CAP) Patient will be complaining of sudden onset of cough, fatigue, and fever PE will show focal rales with auscultation of lung fields Most commonly caused by Streptococcus pneumoniae Confirm dx with CXR or ultrasound (operator-dependent) CURB-65 to stratify risk, inpatient vs. outpatient management Tx: 2019 IDSA guidelines for adult CAP, guidelines for empiric pediatric CAP

__________ _________ is the most accurate blood sampling for true arterial blood gas (ABG) and acid-base determination

Arterial puncture

58 y/o male presents to the clinic, PMH remarkable for COPD on prolonged glucocorticoids and HIV with a CD4 < 50. He complains of fever, pleuritic CP, and hemoptysis. CXR shows multiple nodules, cavitary lesions, and alveolar infiltrates. Ct shows multiple nodules with surrounding GGOs (halo sign). What is your diagnosis and treatment?

Aspergillosis Initial therapy: Voriconazole or lsavuconazole Salvage therapy: Liposomal Amphotericin B

57 y/o male patient with a PMH remarkable for HIV presents to the clinic with acute pnemonia, symptoms of fevers, cough, purulent sputum, pleuritic CP, and hemoptysis. He states he was working as a forest ranger 4 weeks ago in Southeastern US, bordering the Mississippi & Ohio river valleys. What is your diagnosis? A. Pneumocytosis B. Blastomycosis C. Histoplasmosis D. Aspergillosis

B. Blastomycosis Blastomycosis is transmitted by inhalation of spores from soil containing decaying vegetation or rotting wood CXR: patchy alveolar infiltrates of discrete masses; lack of hilar/med adenopathy Gold standard sputum culture confirms your dx Start on Liposomal Amphotericin B for his severe disease; if it was mild/moderate start oral Itraconazole

A 67-year-old man comes to the emergency department because of fever and shortness of breath for the past 6 days. He has associated chills and a cough productive of multiple tablespoons of thick green sputum each morning. He has a history of diabetes and end-stage renal disease requiring regular dialysis which he gets at a center near his home. He lives at home with his wife, has had no recent travel or sick contacts, and no recent hospitalizations. His temperature is 38.5°C (101.3°F), pulse is 100/min, respirations are 22/min, and blood pressure is 100/72 mm Hg. Pulse oximetry on room air shows an oxygen saturation of 92%. He can speak in 3-4 word sentences. Physical examination shows bibasilar crackles on auscultation, and diminished breath sounds at the right middle and right lower lung fields. Which of the following is the most likely diagnosis? A. Aspiration pneumonia B. Community-acquired pneumonia C. Healthcare-associated pneumonia D. Hospital-acquired pneumonia E. Ventilator-associated pneumonia

B. Community-acquired pneumonia Community-acquired pneumonia is defined as any pneumonia occurring outside a hospital settings, while hospital-acquired and ventilator-associated pneumonias occur 48 or more hours after hospitalizations and endotracheal intubation, respectively.

A 67-year-old woman comes to the emergency department because of progressive respiratory distress for 12 hours. She says that she also has chest pain, and is very concerned that she is having a heart attack. She has a history of diabetes mellitus type 2, congestive heart failure, and gout. Current medications include metformin, insulin, cilazapril, allopurinol, and metoprolol. Pulmonary auscultation shows decreased breath sounds in the left lower lobe. A chest X-ray is obtained. Which of the following is the most likely diagnosis?(a) Previous chest x-ray, (b) Today's chest x-ray shown A. Cirrhosis B. Congestive heart failure exacerbation C. Malignant metastases D. Pneumonia E. Pulmonary embolism and infarction

B. Congestive heart failure exacerbation This patient has a left pleural effusion. The costophrenic angle on the left is obscured, as well as the left lower lung field. Considering this patient's past history of congestive heart failure, this is most likely an exacerbation from fluid overload causing a transudative effusion. Transudate: -Total protein ratio in fluid : serum <0.5 -LDH ratio in fluid : serum <0.6 -Specific gravity <1.016 Exudative: -Fluid : serum total protein ratio >0.5 -Fluid : serum LDH ratio >0.6 -Specific gravity >1.016

A 37-year-old male with a past medical history of Crohn disease treated with infliximab is admitted to the hospital after initiation of therapy for active tuberculosis. On day four of hospitalization, the patient reports worsening pain in the left great toe. Temperature is 37.0°C (98.6°F), pulse is 80/min, respirations are 16/min, blood pressure is 120/64 mmHg, and O2 saturation is 93% on room air. Which of the following describes the mechanism of action of the medication likely causing this observed side effect? A. Inhibition of mycolic acid synthesis B. Conversion to pyrazinoic acid and subsequent lowering of environmental pH C. Inhibition of bacterial DNA-dependent RNA synthesis D. Inhibition of the 30s ribosomal subunit E. Inhibition of the enzyme arabinosyl transferase

B. Conversion to pyrazinoic acid and subsequent lowering of environmental pH Pyrazinamide is converted to pyrazinoic acid inside mycobacteria and works best at acidic pH levels. Its side effects include hepatotoxicity and hyperuricemia. This patient was initiated on RIPE therapy (rifampin, isoniazid, pyrazinamide, and ethambutol) for treatment of active tuberculosis. He has developed gouty arthritis and evidence of hepatotoxicity, which are consistent with the side effect profile of pyrazinamide.

A 58-year-old man presents to the clinic with a productive cough, fever, and chills for 3 days. He has a past medical history of excessive alcohol use, tobacco use (20 pack-year history), and hypertension. Which of the following additional history or physical findings would most closely correlate with a diagnosis of community-acquired pneumonia caused by Klebsiella pneumoniae infection? A. Bibasilar crackles B. Currant jelly sputum C. Exposure to contaminated water D. Recent travel to Asia

B. Currant jelly sputum Community-acquired pneumonia refers to pulmonary infection acquired outside of the hospital setting. Common causes of community-acquired pneumonia include viral infection and infection with Streptococcus pneumonia, Haemophilus influenzae, Moraxella catarrhalis, and Staphylococcus aureus. Patients who chronically use alcohol are at increased risk for developing Klebsiella pneumoniae due to aspiration of stomach contents during periods of decreased consciousness. Other patients at increased risk of Klebsiella pneumoniae include patients with severe chronic obstructive pulmonary disease, diabetes mellitus, or those who have been hospitalized. Features of Klebsiella pneumoniae pulmonary infection are similar to those of other causes of community-acquired pneumonia and include fever, chills, tachycardia, dyspnea, pleuritic chest pain, and productive cough. A history of currant jelly sputum is associated with Klebsiella pneumoniae infection. Treatment of community-acquired pneumonia is with broad-spectrum antibiotics, such as azithromycin or doxycycline. Patients infected with Klebsiella pneumoniae are often severely ill and require hospitalization and treatment with a carbapenem antibiotic. Prevention of community-acquired pneumonia is through vaccination against influenza and pneumococcus, as well as smoking cessation

A 67-year-old man presents to the office with worsening orthopnea, dyspnea on exertion, and paroxysmal nocturnal dyspnea. He has a history of left-sided systolic heart failure. Exam reveals a new pulmonic murmur. You suspect pulmonary hypertension. What is the best choice for your initial diagnostic study? A. Chest X-ray B. Echocardiogram C. Electrocardiogram D. Right heart catheterization

B. Echocardiogram Pulmonary hypertension is a disease of increased pulmonary artery pressure and defined as mean pulmonary arterial pressure greater than 25 mm Hg at rest. There are multiple etiologies which are split into classes. Group 1 is pulmonary artery hypertension (PAH), group 2 is pulmonary hypertension due to left heart disease, group 3 is pulmonary hypertension due to chronic lung disease or hypoxemia, group 4 is chronic thromboembolic pulmonary artery hypertension, and group 5 is pulmonary hypertension due to unclear multifactorial mechanisms. A patient with known left heart systolic failure is suspected to be in group 2. Initial symptoms are a result of the inability to increase cardiac output during exercise. The patient will develop exertional dyspnea and fatigue. Symptoms progress as the pulmonary hypertension advances to include angina, exertional syncope, peripheral edema, and anorexia from abdominal ascites associated with right-sided heart failure. Initial exam findings are an loud pulmonic component of the second heart sound (P2) or the second heart sound (S2) may be narrowly split if right ventricular function is maintained. As the right ventricular pressure increases, an A wave jugular venous pulse may develop a fourth heart sound or a heave along the parasternal border. In more severe disease, as the right ventricle fails and develops a right bundle branch block, there may be a tricuspid murmur. In even further advanced disease, a pulmonic regurgitation murmur may develop. The goal of diagnostic studies when pulmonary hypertension is suspected is to confirm diagnosis and determine the etiology. The first step in diagnosis is echocardiogram. The echocardiogram should be interpreted to determine if the left heart failure is significant enough to increase pulmonary artery pressures and create pulmonary hypertension. This also assesses the function and extent of valvular dysfunction. If the echocardiogram shows enough left heart failure to explain the pulmonary hypertension, then no further workup is needed. If there is not a clear justification for the pulmonary hypertension on echocardiogram, then further testing, such as pulmonary function testing, ventilation-perfusion scanning, and overnight oximetry, should be performed. Right heart catheterization (definitive diagnosis) is indicated in patients who still are not found to have a clear explanation of the pulmonary hypertension. Once the etiology is determined, then management is directed at the underlying etiology. Vasodilators may lower pulmonary vascular resistance in some patients. Available vasoactive agents include oral CCBs (first line for primary pulmonary HTN), inhaled phosphodiesterase inhibitors, prostacyclins, and endothelin receptor antagonists.

A 58-year-old obese man comes to the emergency department after becoming severely short of breath after climbing a flight of stairs. A brief history reveals that his shortness of breath is associated with a severe cough and is productive of green sputum. He has had symptoms like these for at least four months out of the year over the past decade, but they have been consistently present for the past 6 months and seem to be worsening in severity. The patient admits to smoking about a pack of cigarettes a week for the past 20 years. He has never received the flu vaccine. He is worried that this could be related to his family history of cardiac issues. He is afebrile, there is no sign of edema, and heart sounds appear to be normal. Which of the following findings is most likely to be observed on physical examination? A. Diffusely increased breath sounds B. Expiration time much longer than inspiration time C. Hypo-resonance on percussion D. Increased tactile fremitus on palpation E. Pulmonary edema

B. Expiration time much longer than inspiration time Chronic Obstructive Pulmonary Disease (COPD) is an obstructive airway disease encompassing both chronic bronchitis and emphysema. Chronic bronchitis is defined clinically as a productive cough for greater than 3 months per year for 2 consecutive years.

Which of the RIPE drugs works by inhibiting mycolic acid synthesis, which interferes with cell wall synthesis? A. RIF B. INH C. PZA

B. INH

Which of the following anti-TB drugs can case increased pyridoxine (B6) excretion which can cause nephrotoxicity? A. RIF B. INH C. EMB D. PZA

B. INH -Prevented by supplementing pyridoxine (vit. B6 daily)

A 56-year-old woman comes to the emergency room because of wheezing and severe shortness of breath for 2 days. Her past medical history includes breast cancer that was treated with paclitaxel over the past several months. Physical examination shows decreased tactile fremitus over the left lower lobe. A chest radiograph is obtained. What's your dx? A. Left lung collapse B. Left pleural effusion C. Right lung collapse D. Right pleural effusion E. Right tension pneumothorax

B. Left pleural effusion Pleural effusions are the presence of fluid in the lungs. These can be caused by a number of causes, including malignancy. Radiographs will often show blunting of the costophrenic angles and the presence of a horizontal fluid line, among other signs.

A 29-year-old man presents to the emergency department with new-onset dyspnea and right-sided chest pain. He is tall and thin, his vital signs are within normal limits except for an oxygen saturation of 92%, and chest X-ray reveals a pneumothorax with a 3.5 cm rim of air in the right pleural space. Which of the following represents the best initial treatment for this patient? A. Chest tube thoracostomy B. Needle aspiration C. Supplemental oxygen and observation D. Video-assisted thoracoscopic surgery

B. Needle aspiration Patients may also be asymptomatic with the pneumothorax found incidentally on chest X-ray. Physical exam findings include decreased chest excursion, hyperresonance to percussion, and decreased breath sounds on the affected side. Patients who have a tension pneumothorax may also present with tachycardia and hypotension, necessitating immediate life-saving thoracostomy. If vital signs are stable, patients with a spontaneous pneumothorax greater than 3 cm or who are symptomatic can be treated with needle aspiration of the pneumothorax. The aspiration is performed with an 18 gauge needle in the second or third intercostal space at the midclavicular line. A catheter is inserted, and air is removed completely. If there appears to be a persistent leak, tube thoracostomy and thoracoscopy with pleurodesis are required. A pneumothorax can be definitively diagnosed with a chest X-ray, and serial chest X-rays are also used to gauge treatment effectiveness and lung reinflation. Recurrence of spontaneous pneumothorax can be diminished with smoking cessation and pleurodesis.

A 60-year-old man comes to the emergency department because of progressive shortness of breath and chest pain for 3 weeks. His medical history includes diabetes mellitus, hyperlipidemia, and a myocardial infarction 6 years ago. He is in moderate respiratory distress. Pulmonary auscultation shows decreased breath sounds throughout both lung fields. A chest x-ray is obtained and confirms a pleural effusion. Cardiac auscultation demonstrates a heart sound occurring after S2. Thoracentesis is performed, and 800 mL of fluid is removed. Which of the following laboratory results is most likely to be found? A. Pleural fluid LDH/serum LDH ratio of 0.85 B. Pleural fluid protein/serum protein ratio of 0.25 C. Pleural fluid protein/serum protein ratio of 1.2 D. Pleural fluid that has an LDH which is 75% of the laboratories upper limit of normal for serum LDH E. Pleural fluid protein/serum protein ratio of 0.55

B. Pleural fluid protein/serum protein ratio of 0.25 Congestive heart failure leads a transudative effusion, which would not fulfill any of the Light's criteria. Meeting of any of these criteria indicates the presence of an exudative effusion. The criteria are as follows: -Pleural fluid protein/serum protein ratio greater than 0.5, or -Pleural fluid LDH/serum LDH ratio greater than 0.6, or -Pleural fluid LDH greater than two-thirds the upper limits of the laboratory's normal serum LDH.

Which of the following is NOT a cause of diaphragmatic elevation? A. Lobectomy B. Pneumothorax C. Phrenic nerve palsy D. Ascites

B. Pneumothorax A large pneumothorax may flatten the diaphragm. The other conditions may cause diaphragmatic elevation

Which hemidiaphragm is usually higher? A. Left B. Right

B. Right The right hemidiaphragm is usually 1cm higher than the left, due to the presence of the liver.

A 35-year-old man comes to the office because of a 5-day history of a productive cough, shaking chills, and fever. His medical history is relevant for occasional smoking, bipolar disorder, and a splenectomy when he was 10. Upon further interrogation, the patient reports having a "rusty" sputum whenever he coughs. On physical examination, auscultatory findings reveal inspiratory crackles on inspiration and increased tactile and vocal fremitus. His temperature is 39.7°C (102.2°F), pulse is 106/min, respirations are 24/min, blood pressure is 120/80 mmHg, oximetry on room air shows an oxygen saturation of 97%. A conventional chest radiograph is obtained and shown below. This patient is at increased risk for developing which of the following? A. Pulmonary abscess B. Sepsis C. Pleural effusion D. Acute respiratory distress syndrome E. Bronchiectasis

B. Sepsis The patient in the vignette is experiencing a classic presentation of bacterial pneumonia. The most key part of the diagnosis here is the clinical presentation (i.e., fever, shaking chills, right-middle-lobe consolidation, productive cough or "rusty sputum"). Bacterial pneumonia is a lung infection that results from acute inflammation of the alveoli caused by bacteria. The most common cause of bacterial pneumonia is Streptococcus pneumoniae. Streptococcus pneumoniae is an encapsulated, gram positive, lancet-shaped diplococci. On blood agar, they are alpha-hemolytic, catalase negative, and optochin-susceptible. The capsule of Streptococcus pneumoniae makes asplenic patients be at a considerably higher risk to develop sepsis.

What are Kerley B lines due to? A. Fluid in the deep septae B. Thicking of the interlobular septae C. Fluid within the alveoli D. Pulmonary scarring

B. Thicking of the interlobular septae Kerley B lines, a sign of pulmonary edema (amongst other conditions), occurs due to interlobular septal thickening.

A 28-year-old woman presents to the clinic with skin lesions and a chronic cough. Multiple flesh-colored papules are noted on her face and neck. Chest X-ray reveals bilateral hilar lymphadenopathy and reticular opacities in bilateral lung apices. Which of the following diagnostic choices would be most useful to establish the diagnosis of sarcoidosis in this patient? Biopsy of a skin lesion Pulmonary function tests Serum angiotensin-converting enzyme level Serum calcium level

Biopsy of a skin lesion Sarcoidosis Labs will show hypercalcemia and elevated serum ACE CXR will show bilateral hilar adenopathy Biopsy will show noncaseating granulomas Treatment is steroids Comments: Lupus pernio (chronic, violaceous, raised plaques and nodules commonly found on the cheeks, nose, and around the eyes) is pathognomonic for sarcoidosis and is the most specific physical exam finding in this disease

_____________ is transmitted through inhalation of spores from soil containing decaying vegitation or rotting wood.

Blastomycosis

__________ is a respiratory drug used to treat pulmonary hypertension by competitively antagonizing endothelin, thereby decreasing pulmonary vascular resistance.

Bosentan

What duration of treatment with isoniazid is needed for latent TB infections? A. 2 months B. 6 months C. 9 months D. 4 months

C. 9 months

A 41-year-old woman comes to the hospital because of uncontrollable fits of coughing, and trouble breathing over the past week. Due to the recent Ebola scare, she is quickly quarantined from other patients and put in a special room. She has a cough productive of clear sputum and denies night sweats. Her temperature is 37°C (98.6°F), pulse is 70/min, and respirations are 20/min. Pulse oximetry shows oxygen saturation of 95% on room air. The rest of the physical examination is normal except for minimal rales at the posterior and inferior lung margins. Which of the following is the most appropriate step for acute management? A. Administering a fluoroquinolone B. Administering a macrolide C. Administering albuterol and antitussives D. Offering oxygen therapy E. Performing a sputum gram stain

C. Administering albuterol and antitussives Acute bronchitis in adults is of viral origin and should not be treated with antibiotics. Management can be reassurance or symptomatic treatment with bronchodilators or antitussives.

You are treating a 54 y/o male with PMH remarkable for HTN, DM, and asthma. He is diagnosed with community acquired pneumonia and should be treated with ABx. What do you start him on? A. Azithromycin + Cefipime B. Gentamicin + Cefepime C. Amox/clav + azithromycin

C. Amox/clav with azithromycin

You are babysitting your 4 year old nephew when they beging to grab their throat and become unable to speak or cough while eating dinner. What is your next step? A. Finger sweep B. Let them cough it up themselves C. Chest thrusts, back blows, abdominal thrusts

C. Chest thrusts, back blows, abdominal thrusts *Blind finger sweep is contraindicated in children due to risk of pushing it deeper

35 y/o male patient with a PMH remarkable for HIV presents to the clinic with acute pneumonia. He has symptoms of ever, fatigue, cough, pleuritic chest pain, and night sweats. He states that 2 weeks ago he was on a camping trip in the desert of San Joaquin Valley. What is your diagnosis? A. Pneumocytosis B. Histoplasmosis C. Coccidioidomycosis D. Aspergillosis

C. Coccidioidomycosis For the dx: CXR: unilateral dense infiltrate in UL with ipsilateral hilar/med adenopathy Serological testing for IgM and IgG (has highest sens. and spec.) Cytology or bx with staining Treatment: Severe disease, treat with oral fluconazole (if stable), liposomal amphotericin B (if unstable) Mild-moderate disease: no treatment; if RFs or IC, oral fluconazole

An 82-year-old Caucasian man comes to the emergency department because of a 1-week history of fever and pleuritic pain. His medical history is relevant for type II diabetes mellitus, systemic sclerosis, and a recent hospitalization due to community-acquired pneumonia. On physical examination the patient has lateral chest wall swelling and tenderness. Auscultatory findings reveal decreased respiratory sounds and increased right-side vocal fremitus. His temperature is 39.7°C (103.4°F), pulse is 100/min, respirations are 70/min, blood pressure is 130/80 mmHg, and pulse oximetry on room air shows an oxygen saturation of 93%. A chest radiograph shows a unilateral, biconvex, and encapsulated fluid collection on the right chest wall. A pleural drainage is performed and shown below. Which of the following is the most likely cause of this patient's current condition? A. Mesothelioma B. Aspergilloma C. Empyema D. Lung adenocarcinoma E. Pleural effusion

C. Empyema Empyema is a pleural cavity bacterial infection predominantly caused as a complication of pneumonia that results in the accumulation and collection of pus. The mainstay therapy is to treat the underlying infection and pleural drainage of the collection.

A 55-year-old Caucasian woman comes to the office because of a 1-week history of a productive cough and shortness of breath. Her medical history is relevant for type II diabetes mellitus, high blood pressure, and major depressive disorder. She currently takes metformin, telmisartan, and escitalopram. Upon further interrogation, the patient reports having yellowish sputum whenever she coughs. On physical examination, the patient looks pale and has shaking chills. Auscultatory findings reveal inspiratory crackles on inspiration and increased tactile and vocal fremitus. Her temperature is 39.7°C (102.2°F), pulse is 122/min, respirations are 27/min, blood pressure is 130/61 mmHg, oximetry on room air shows an oxygen saturation of 96%. A conventional chest radiograph is obtained and shown below. Which of the following best describes the causal organism responsible for this patient's condition? A. Encapsulated, gram positive, diplococci, bacitracin sensitive B. Unencapsulated, gram positive, positive acid-fast stain C. Encapsulated, gram positive, diplococci, optochin sensitive D. Encapsulated, gram negative, rod, oxidase negative E. Unencapsulated, gram negative, coccobacillary, bacitracin sensitive

C. Encapsulated, gram positive, diplococci, optochin sensitive Community-acquired pneumonia (CAP) is a lower tract respiratory infection that is most commonly caused by Streptococcus pneumoniae, an encapsulated, gram positive, diplococci, and optochin sensitive microorganism.

A 34-year-old male with a past medical medical history of HIV, type I diabetes mellitus, and polysubstance abuse presents to the emergency department with worsening cough and night-sweats. He has been unable to sleep at night due to the cough. The patient has occasionally noticed blood in his sputum. He is currently undomiciled, drinks several beers daily, and smokes one pack of cigarettes per day. The patient's temperature is 37.2°C (99.0°F), pulse is 94/min, respirations are 16/min, blood pressure is 136/74 mmHg, and O2 saturation is 94% on room air. A screening PPD test is positive. Which of the following is the strongest risk factor for the development of this patient's disease process? A. Age B. Diabetes mellitus C. HIV D. Homelessness E. Active smoking

C. HIV HIV is the strongest predisposing risk factor for the development of active tuberculosis. Other major risk factors include substance abuse, extremes of age, comorbid conditions, and exposure to at risk populations.

A 16-year-old male presents to the primary care physician with fevers, cough, and night sweats. He is accompanied by his aunt, who reports the patient has been coughing non-stop at night and has even had small specks of blood in his sputum. The patient is otherwise healthy and recently moved to the United States from the Philippines. Temperature is 38.3 °C (101°F), pulse is 86/min, respirations are 18/min, blood pressure is 123/74 mmHg, and O2 saturation is 95% on room air. Physical exam reveals a malnourished patient and is notable for faint rales over the right lower lung field. Which of the following radiographic features is most indicative of this patient's primary infectious process? A. Unilateral pleural effusion B. Bilateral pulmonary infiltrates C. Hilar lymphadenopathy and a primary focus of infection in the mid-lower lobes D. Fibrocaseous cavitary lesion in the upper lobes E. Cavitary caseating lesion

C. Hilar lymphadenopathy and a primary focus of infection in the mid-lower lobes This pediatric patient who recently immigrated from the Philippines presents with night sweats, fevers, and cough. Physical exam is notable for a cachexia, malnourishment, and diminished breath sounds over the right lower lung. Together, the history and exam findings are most concerning for primary pulmonary tuberculosis, of which the most common radiographic feature is hilar lymphadenopathy and a Ghon focus in the mid-lower lobes.

An 82-year-old man was admitted to the ICU 14 days ago for an acute coronary event. His clinical course required that he be ventilated. He has now developed hypoxia, fever, and pulmonary infiltrates. Which of the following antimicrobials represent the best treatment plan for this patient's nosocomial pneumonia? A. Azithromycin and levofloxacin B. Ceftriaxone C. Imipenem, gentamicin, and vancomycin D. Piperacillin-tazobactam

C. Imipenem, gentamicin, and vancomycin Azithromycin and levofloxacin (A) are not a good choice for this patient. Azithromycin would be a good choice for a patient with community-acquired pneumonia, and levofloxacin would be a good choice for a patient with nosocomial pneumonia who was not at risk for infection with multidrug-resistant organisms. Ceftriaxone (B) is a third-generation cephalosporin and is indicated for the treatment of acute bacterial otitis media, abdominal infections, pelvic inflammatory disease, gonococcal infections, surgical prophylaxis, meningitis, pyelonephritis, and prosthetic joint infection. Ceftriaxone has poor activity against Pseudomonas, making it an inferior choice for treatment of nosocomial pneumonia. Piperacillin-tazobactam (D) is a beta-lactam antibiotic with good activity against gram-negative bacilli. It is a good first-line choice for patients with nosocomial pneumonia who have no risk factors for multidrug-resistant bacterial infections.

34 y/o male patient with an unremarkable PMH presents to the ER with dyspnea, uneven chest expansion, and chest pain. He is a current, everyday smoker. On physical exam, you hear hyperresonance with percussion and diminished breath sounds on the affected side. CXR confirms a pneumothorax. What are your treatment options? A. IV antibiotics B. Watchful waiting C. Oxygen, simple needle aspiration D. Needle decompression

C. Oxygen, simple needle aspiration This patient has primary pneumothorax. He has risk factors that put him at high risk of rupture of small pulmonary blebs. These patients are typically aged 18-40 years old, tall, thin, and often smokers. This is performed at the 2nd intercostal space, midclavicular line.

A 58-year-old man goes to the emergency department for evaluation of fever and dry cough for the past week. He has also become progressively more dyspneic, and walking a single block makes him short of breath. He underwent a liver transplantation 4-months ago for treatment of autoimmune hepatitis and is currently taking several immunosuppressant drugs. Temperature is 38.5°C (101.3°F), pulse is 104/min, respirations are 20/min, blood pressure is 122/88 mmHg, and oxygen saturation is 89% on room air. Lung auscultation reveals bilateral crackles and rhonchi. Chest radiograph reveals bilateral, diffuse interstitial infiltrates extending from the perihilar region. Leukocyte count is 13,000/mm3 and serum lactate dehydrogenase level is 460 U/L. Further testing is most likely to reveal which of the following findings? A. Detection of bacterial antigen in urine B. Growth of bacteria on cholesterol-rich agar C. Presence of organisms in sputum that stain positive for methenamine silver D. Identification of Gram-positive diplococci on sputum analysis E. Visualization of acid-fast organisms on sputum microscopy

C. Presence of organisms in sputum that stain positive for methenamine silver Pneumocystis jirovecii pneumonia (PCP) is an opportunistic fungal lung infection occurring almost exclusively in immunocompromised or immunosuppressed individuals. Patients typically present with fever, nonproductive cough, and dyspnea. Patients usually have elevated lactate dehydrogenase levels, and diagnosis can be confirmed by visualizing P. jirovecii after methenamine silver staining.

A 76-year-old man presents with exertional dyspnea, fatigue, and a cough. History reveals a 45 pack-year smoking history and resultant COPD. An echocardiogram reveals right ventricular wall thickening with paradoxical motion of the interventricular septum during systole. You make the diagnosis of cor pulmonale. Which of the following is the most likely underlying cause? A. Coxsackievirus B. Hypertrophic cardiomyopathy C. Pulmonary hypertension D. Systemic hypertension

C. Pulmonary hypertension disease, such as pulmonary hypertension resulting from COPD. The most common cause of chronic cor pulmonale is COPD and the most common cause of acute cor pulmonale is pulmonary embolism. The symptoms of cor pulmonale may be very subtle but may include exertional dyspnea, fatigue, cough, tachypnea, and anginal chest pain. Physical exam may reveal an increased chest diameter, labored breathing with retractions, cyanosis, digital clubbing and wheezes or crackles on lung auscultation may be heard, which reflect the underlying lung disease. Signs of right ventricular failure such as hepatomegaly, edema, JVD may also be noted. Several tests are used to diagnose cor pulmonale, including routine lab work, chest radiography, ECG, echocardiography, and right heart catheterization. CXR may reveal enlargement of the RA, RV, and pulmonary arteries. ECG may show right axis deviation, P pulmonale, and RVH. Echocardiogram may reveal elevated pulmonary artery systolic pressures, and altered right-sided ventricle structure or dysfunction. Right heart catheterization is the gold standard test to confirm the diagnosis of pulmonary hypertension. This test is invasive, so not every patient requires this procedure. Once the diagnosis of cor pulmonale is made, the underlying lung pathology must be diagnosed and addressed as well. Treatment of cor pulmonale is to treat the underlying lung condition.

A 71-year-old woman presents to the clinic for dyspnea on exertion and pedal edema. She has a 25 pack-year smoking history. On physical exam, a loud second heart sound is noted, with a widely split S2, S4, and a holosystolic murmur that is loudest during inspiration at the third intercostal space to the left of the sternum. ECG reveals right bundle branch block. Which of the following diagnoses is most likely? A. Aortic valve stenosis with left heart failure B. Myocardial infarction with left heart failure C. Pulmonary hypertension with right heart failure D. Pulmonic valve stenosis with right heart failure

C. Pulmonary hypertension with right heart failure Pulmonary Hypertension Mean PA pressure of 25 mm Hg or more Patient complaining of exertional dyspnea, syncope, and/or fatigue Physical exam: Initially: often cyanosis, usually increased intensity of the second heart sound As PH progresses - signs of RV failure JVP is typically elevated, prominent a wave, prominent v wave (seen often in severe RV failure) A right-sided third or fourth heart sound (ie, a gallop) CXR: tapering PAs, RVH Echocardiography is often diagnostic (enlarged RV and RA) Management includes: O2 sat is greater than 90 percent, refer to pulm/cardio specialist Types: Group 1: PAH Group 2: Left heart disease Group 3: Lung Disease Group 4: Chronic thromboembolic pulmonary HTN Group 5: Multifactorial/Unknown

A 45-year-old man comes to the emergency department with left lower quadrant pain, fever. This is the 4th episode this year. A diagnosis of recurrent diverticulitis is made and a left colectomy is scheduled for tomorrow. During anesthesia, an arterial blood gas is drawn and shows:pH: 7.3PaCO2: 70 mm HgHCO3-: 30 mEq/L. Which of the following best describes the patient's acid-base disorder? A. Metabolic acidosis with compensation B. Metabolic alkalosis with incomplete compensation C. Respiratory acidosis with compensation D. Respiratory acidosis without compensation E. Respiratory alkalosis with incomplete compensation

C. Respiratory acidosis with compensation Respiratory acidosis is defined as an acidotic pH (< 7.35) with increased pCO2 (typically > 40 mmHg). To assess compensation, there should be a rise of about 1 mEq/L of HCO3- for every 10 mmHg rise in CO2-.

A 5-year-old girl is brought to the emergency department by her parents because of a sore throat and difficulty swallowing that began 3 days ago. Her temperature is 39.2 °C (102.7 °F), pulse is 92/minute, respiration rate 22/minute, and blood pressure is 100/73 mm Hg. Her O2 saturation of 98% on room air. Physical examination shows an erythematous oropharynx, bilateral tonisillar exudates, and cervical lymphadenopathy. During the examination, the patient resists moving her head and begins drooling. A lateral cervical spine X-ray shows a widened pre-vertebral space. Which of the following is the most likely diagnosis? A. Epiglottitis B. Laryngotracheobronchitis C. Retropharyngeal abscess D. Meningitis E. Peritonsillar abscess

C. Retropharyngeal abscess Dysphagia is common in an upper respiratory tract infection, but if the patient displays torticollis, change in voice, drooling, and pain it may be due to a retropharyngeal abscess.

Which of the following diagnostic tests is definitive for TB? A. Chest x-ray B. Mantoux test C. Sputum culture D. Tuberculin test

C. Sputum Culture The sputum culture for Mycobacterium tuberculosis is the only method of confirming the diagnosis. Lesions in the lung may not be big enough to be seen on x-ray. Skin tests may be falsely positive or falsely negative.

A 75-year-old woman presents to the emergency room for progressive shortness of breath. History is positive for a recent bout of pneumonia for which she had been prescribed appropriate antibiotics. The patient admits that she has been mostly non-compliant with taking the medication. A chest X-ray is performed and is displayed below. Which of the following interventions would most likely give you the etiology of the patient's pathology? A. PET B. CT of the chest C. Thoracentesis D. CBC E. Pericardiocentesis

C. Thoracentesis Unilateral pleural effusions have several causes including malignancy, cylothorax, hemorrhage, and infection. Radiologically, it can be identified by radiograph. Typical findings include the presence of a horizontal fluid line within one or more fissures as well as opacity in the region of the effusion. A thoracentesis should be performed to determine the underlying etiology. During this procedure a needle is advanced into the posterior chest and a sample of the effusion is taken. Cytology and labs will be able to determine the cause of the effusion. Thoracentesis can also be therapeutic, as draining the fluid from the chest cavity can decrease a patients respiratory distress.

A 65 year old man presents to the emergency department with shortness of breath and chest pain which has been getting progressively worse over three weeks. His vitals are HR 110, RR 28, BP 130/85, T 98.3, and SpO2 92% on room air. Medical history is significant for diabetes mellitus, hyperlipidemia, and a heart attack 5 years ago. Physical examination shows a patient in moderate respiratory distress. Breath sounds are decreased in the lower half of both lung fields, and there is decreased tactile fremitus in the lower third of both lung fields. Auscultation of the heart reveals a regular rhythm and a protodiastolic gallop. He has jugular venous distention to his earlobe when he reclines to 30°. Which of the following is the most consistent with his presenting symptoms and physical examination? A. Heart failure due to a non-compliant right ventricle B. Exudative effusion between visceral and parietal pleura C. Transudative effusion between visceral and parietal pleura D. Tension pneumothorax E. Pericardial tamponade

C. Transudative effusion between visceral and parietal pleura The patient has a pleural effusion from a first presentation of congestive heart failure which is most likely due to an ischemic cardiomyopathy, given his history of a heart attack. Failure on the left side of the heart causes pooling of blood in the pulmonary circulation. The increase in hydrostatic pressure eventually causes fluid to shift into the pleural space, causing a transudative pleural effusion. The pleural effusion is responsible in part for the patients respiratory distress, chest pain, and physical exam findings. The protodiastolic gallop - a.k.a an S3, or a third- heart sound - is a sign of vascular congestion and fluid overload, as is the patient's elevated JVP.

A (CXR/CT) ________ has a sensitivity of almost 100% in identifying foreign bodies. It can help identify the exact location of the foreign body.

CT scan Main disadvantage: ionizing radiation, delay to bronchoscopy

A 23-year-old man presents to the emergency department via emergency medical services following a motor vehicle accident. He was the restrained driver in a collision at 55 mph. His condition is currently stable and you are assessing for any internal injuries to the chest. Which of the following is the first imaging modality you should perform? Chest computed tomography Chest radiograph Electrocardiogram Thorax magnetic resonance imaging

Chest radiograph A chest radiograph is the initial choice for patients that warrant diagnostic imaging unless the patient is unstable enough to require emergent surgery or emergent computed tomography. Chest radiograph is quick, inexpensive, non-invasive, and in many cases can reveal the etiology being sought. Radiographs should be reviewed for signs of pneumothorax, hemothorax, widened mediastinum suggesting aortic injury, and mediastinal shift. A widened mediastinum or abnormal aortic contour are suggestive of blunt aortic injury.

______________________ is the second most common cause of atypical pneumonia in young adults.

Chlamydia pneumoniae

Two species of Chlamydiae, Chlamydia ______________ and Chlamydia __________, cause atypical pneumonia; transmitted by aerosol.

Chlamydia pneumoniae and Chlamydia psittaci

________________ has an avian reservoir, namely parrots, and causes atypical pneumonia.

Chlamydia psittaci

A 53-year-old man presents with a cough. He states that the cough has been getting worse over the last three months, and now he is starting to have blood in his sputum. He denies fevers. He has a 60 pack-year cigarette smoking history. Which of the following is the most likely diagnosis? Chronic bronchitis Lung malignancy Pneumonia Pulmonary embolism

Chronic bronchitis Hemoptysis Diastolic murmur: mitral stenosis Sudden SOB, CP: PE Trauma: pulmonary contusion Immunodeficiency, immigrant: TB Hx of TB or sarcoidosis: aspergilloma Renal dysfunction: Goodpasture's syndrome or Granulomatosis with Polyangiitis (GPA, Wegener's) Hx of tobacco use, weight loss: malignancy Massive hemoptysis: ≥ 100 mL/hour or ≥ 500 mL over 24-hours Massive hemoptysis rx: patient in bleeding side down position, mainstem bronchus intubation

A 70-year-old man presents with a gradual onset of dyspnea on exertion and a nonproductive cough. He reports a 30 pack-year history. He is otherwise healthy. Laboratory studies are normal at this time. Chest radiography reveals reticular opacities in the lower lung. High resolution computed tomography shows subpleural honeycombing. Which of the following physical examination findings is most likely to be present in this patient? Clubbing Fever Janeway lesions Osler nodes

Clubbing Idiopathic Pulmonary Fibrosis Patient will be a male With a history of smoking or environmental exposure Complaining of chronic dry cough and dyspnea Labs will show restrictive characteristics - decreased FVC and FEV1 but a near normal FEV1/FVC ratio CXR/CT will show "honeycombing" Management includes O2, pulmonary rehabilitation, antifibrotic therapy (pirfenidone and nintedanib), possible lung transplant

A 50-year-old man with a history of a kidney transplant presents to the clinic with a 10-day history of cough, fever, and pleuritic chest pain. He recently visited Arizona. Chest X-ray shows a unilateral infiltrate with hilar adenopathy. Which of the following diagnoses is most likely in this patient? Blastomycosis Candidiasis Coccidiomycosis Histoplasmosis

Coccidiomycosis Coccidioidomycosis Presents with fever, cough, chest pain, fatigue, dyspnea, rash, headache, arthralgias, myalgias Fungal infection SW United States Dust exposure Treat with fluconazole or itraconazole

Your patient was recently infected with influenza about 1 month ago and now is showing symptoms of hemoptysis, severe hypoxemia, high fever, leukopenia, hypotension, and cyanosis. On CXR, you see multi-lobular cavitating alveolar infiltrates. What should you be concerned about?

Community acquired PNA - MRSA This diagnosis is rare, but this patient shows classic symptoms and exam findings of a patient with CA-MRSA. They also have a recent influenza infection which is another risk factor.

In a (compressive/resorbtion) _______________ atelectasis, the trachea shifts AWAY from the affected side.

Compressive

A 62-year-old man reports to the emergency department for evaluation of his progressive shortness of breath. Diagnostic evaluation reveals a transudative pleural effusion. Which of the following comorbid conditions is most likely present? Congestive heart failure Infection Malignancy Pancreatitis

Congestive heart failure The most common cause of this type of pleural effusion is congestive heart failure. Patients with congestive heart failure are in a fluid overloaded state and, due to excessive hydrostatic pressure within the vessels and cardiac system, fluid moves from this high-pressure system to the low-pressure pleural space. Diseased organs or organ systems, particularly lung and pleural inflammation or impaired lymphatic drainage, more commonly cause exudative pleural effusions. Malignancy, infection, immunologic responses, and connective tissue disease are among the various mechanisms by which exudative pleural effusions arise.

__________________ is a cardiac complication of pulmonary hypertension that involves right ventricular failure, presenting as jugular venous distension and hepatomegaly.

Cor pulmonale

Which of the following is consistent with the classic triad of foreign body aspiration in a young child? Choking, coughing, and wheezing Coughing, diminished breath sounds, and wheezing Hemoptysis, retractions, and stridor Mediastinal shift, retractions, and stridor

Coughing, diminished breath sounds, and wheezing Foreign Body Aspiration Patient will be a child Complaining of a sudden episode of coughing, wheezing, or stridor Comments: most common site is the right main bronchus

A 72-year-old man presents to the clinic complaining of four days of productive cough, shortness of breath, and fever. Which of the following physical exam findings would be most consistent with a diagnosis of community-acquired pneumonia? Bilateral wheezes on chest auscultation Crackles on chest auscultation Decreased tactile fremitus on chest palpation Hyperresonance on chest percussion

Crackles on chest auscultation Community-acquired pneumonia refers to lung parenchymal infection, either bacterial or viral in nature, acquired in a setting outside the hospital. The major risk factors for community-acquired pneumonia include advanced age, chronic illness, crowded living conditions, smoking, alcohol abuse, aspiration, and viral respiratory infection. Signs and symptoms of community-acquired pneumonia include fever, chills, productive cough, dyspnea, pleuritic chest pain, and tachycardia. Physical exam may reveal increased tactile fremitus, diminished lung sounds, bronchial lung sounds, crackles, or egophony. Common bacterial pathogens responsible for community-acquired pneumonia include Streptococcus pneumoniae, Haemophilus influenzae, Moraxella catarrhalis, and Staphylococcus aureus. Common viral pathogens include influenza A and B, coronavirus, rhinovirus, adenovirus, and parainfluenza virus. Definitive diagnosis of pneumonia is made with chest radiography, which reveals patchy lung infiltrate or lung consolidation. Treatment of community-acquired pneumonia in patients with low risk for resistance and no previous antibiotic use in the preceding three months is with doxycycline or a macrolide. For patients who have used an antibiotic in the preceding three months or who have comorbidities, a dual therapy regimen with amoxicillin-clavulanate and a macrolide or doxycycline is recommended. For patients who live in an area with known high resistance to macrolide antibiotics, doxycycline is the preferred agent. Avoiding crowds, receiving the pneumococcal and influenza vaccines, and smoking cessation are all preventive measures.

__________ is a viral infection causing laryngotracheobronchitis.

Croup -Edema and inflammation of the narrowed pediatric airway below the glottis -Seal like barking cough -Can cause inc. work of breathing -Typically 3 mos to 3 years

A 25-year-old man is brought to the emergency department because of a motor vehicle crash at approximately 45mph 25 minutes ago. On admission, he states that he has generalized pain and dyspnea. Examination shows he is alert and tachypneic, with unilaterally decreased breath sounds and dullness to percussion on the left side. His vital signs and the rest of his trauma-specific physical examination are otherwise unremarkable. Which of the following is the most appropriate initial diagnostic test in this patient? A. An ultrasound B. A diagnostic thoracentesis C. Diagnostic thoracotomy D. A chest x-ray E. A chest CT

D. A chest x-ray Hemothorax is a type of pleural effusion in which blood accumulates in the pleural cavity. This excess fluid can interfere with normal breathing by limiting the expansion of the lungs. Upright chest radiography is the ideal primary diagnostic study in the evaluation of hemothorax.

Which of the following is our first line of defense against pH shift? A. Respiratory mechanism (CO2 excretion) B. Renal mechanism (H+ excretion) C. Phosphate buffer system D. Bicarbonate buffer system E. Protein buffer system

D. Bicarbonate buffer system

A 52-year-old man comes to the office due to 3 days of progressive dyspnea and purulent sputum production. The patient takes albuterol and tiotropium bromide for moderate chronic obstructive pulmonary disease. His medical history is relevant for a 40 pack-year smoking history, type II diabetes mellitus, hyperlipidemia, and coronary artery stenting 2 years ago. Physical exam shows barrel shaped chest, inspiratory crackles, hepatojugular reflux, pulsus paradoxus, and ventricular gallop. His temperature is 38.1°C (100.5°F), the pulse is 130/min, respirations are 28/min, blood pressure is 130/84 mmHg, and pulse oximetry on room air shows an oxygen saturation of 86%. This patient most likely has which of the following conditions? A. Diastolic heart failure B. High-output heart failure C. Systolic heart failure D. Cor pulmonale E. Left-sided heart failure

D. Cor pulmonale This patient who developed signs and symptoms of right heart failure (progressive dyspnea, hepatojugular reflux, pulsus paradoxus, and ventricular gallop) with a history of chronic obstructive pulmonary disease, likely has cor pulmonale. Cor pulmonale is characterized by a right ventricular afterload that causes right ventricular failure and ventricular hypertrophy. This condition is usually derived from pulmonary hypertension that is associated with diseases of the lung, such as chronic obstructive pulmonary disease or sleep obstructive apnea.

Your patient who is taking EMB for TB treatment wants to know what adverse reactions may come from taking this drug. What do you advise them? A. Drug is capable of causing gout and stomach upset B. Drug is capable of causing discolration of body fluids (orange urine, sweat, tears) C. Drug is capable of hepatic toxicity D. Drug is capable of causing eye damage (blurred/changed vision, changed color vision).

D. Drug is capable of causing eye damage (blurred/changed vision, changed color vision). PZA is the drug that may cause (A) RIF is the drug that may cause (B) INH, PZA, RIF are the drugs capable of causing (C)

Which of the following drugs works by inhibiting arabinosyl transferase enzyme, which is responsible for synthesis of a cell wall component of MTB? A. PZA B. RIF C. INH D. EMB

D. EMB

Which of the following is often a complication of pneumonia where bacteria escape into the pleural space? A. Pneumothorax B. Transudative effusion C. Malignant pleural effusion D. Empyema

D. Empyema Pneumonia -> parapneumonic effusion -> complicated parapneumonic effusion -> empyema Empyema is an infection & collection of pus in the pleural space May also be caused by trauma, esophageal rupture, complication of lung surgery, thoracentesis, and chest tube placement. This is technically an exudate effusion

A 42-year-old male with a past medical history of HIV and hypertension presents to the emergency department with worsening anorexia, cough, and night sweats. The patient has lost 15 pounds over the past 2 weeks. He smokes one pack of cigarettes per day and is non-compliant with his HIV medications. Temperature is 37.2°C (99.0°F), pulse is 94/min, respirations are 16/min, blood pressure is 156/74 mmHg, and O2 saturation is 94% on room air. Physical exam reveals a thin, cachectic-appearing male with scattered, coarse breath sounds. Mantoux skin test is positive. Which of the following characteristics best describes the pathogen responsible for this patient's symptoms? A. Gram positive weakly acid fast aerobe B. Gram negative aerobic diplococci C. Gram positive spore forming obligate anaerobic rods D. Gram positive acid fast aerobe E. Gram positive intracellular rod

D. Gram positive acid fast aerobe Mycobacterium tuberculosis is a gram positive acid fast aerobe. It is transmitted via droplet nuclei including airborne particles. It has a waxy cell wall made up of mycolic acid and stains red with Ziehl-Neelsen stain.

A 66 year-old woman comes to the emergency department because of worsening confusion for the past day. Past medical history is significant for Parkinson disease and diabetes mellitus type 2. She is admitted for observation and returns to baseline mentation after 3 days, but on the third day she begins complaining of worsening shortness of breath and cough productive of white sputum. The cough is also associated with chest pain during deep inhalation. Associated symptoms include sinus congestion and headache. Her temperature is 38.3°C (101.3°F), pulse is 92/min, respirations are 22/min, blood pressure is 140/90 mm Hg, and oxygen saturation is 94% on 2L O2 via nasal cannula. Labs and imaging are pending. At this time, which of the following is the most likely diagnosis? A. Atypical pneumonia B. Community-acquired pneumonia C. Healthcare-associated pneumonia D. Hospital-acquired pneumonia E. Lobar pneumonia

D. Hospital-acquired pneumonia Hospital-acquired pneumonia is defined as pneumonia contracted at least 48 hours after being admitted into a healthcare setting (hospital, long-term care, etc.).

A 26-year-old man presents to the emergency department during the winter with sudden-onset fever and nonproductive cough. He also reports headache, chills, and myalgias. Vital signs are T 103.0°F, HR 115 bpm, BP 116/76 mm Hg, RR 20/min, and oxygen saturation 98% on room air. The heart has a regular rate and rhythm without murmurs, and the lungs are clear to auscultation bilaterally. Which of the following is the most likely diagnosis? A. Common cold B. Community-acquired pneumonia C. Infective endocarditis D. Influenza

D. Influenza Influenza Patient will be complaining of sudden onset of fever, headache, cough, myalgia, sore throat, fatigue Diagnosis is made clinically. Can be confirmed with reverse transcription-polymerase chain reaction (RT-PCR) or viral culture Treatment is mainly supportive or oseltamivir for high-risk patients Start oseltamivir as early as possible and after 48 hours, it may not confer any benefit Comments: Most common cause of viral pneumonia In adults. New vaccine needed yearly Monitor patients for post-influenza pneumonia

A 12-year-old boy comes with his mother to the clinic for an annual check-up. His past medical history is significant for a cardiac transplant 1 year ago. His mother indicates that he has been well for the past year without complications. He currently takes tacrolimus and mycophenolate mofetil. Physical examination is unremarkable. Which of the following immunizations would be contraindicated in this patient? A. Hepatitis A vaccine B. Human papillomavirus vaccine C. Intramuscular influenza vaccine D. Intranasal influenza vaccine E. Salk (injectable) polio vaccine

D. Intranasal influenza vaccine Live vaccines are absolutely contraindicated in immunosuppressed individuals such as those who are taking two immunosuppressant drugs to prevent graft rejection. The intramuscular vaccine is preferred for patients in this situation.

A 39-year-old woman comes to the office for her routine cardiology examination. She was diagnosed with limited cutaneous scleroderma two years ago, for which she is currently undergoing treatment with methotrexate and nifedipine. A portion of her ECG is shown below. Which of the following abnormalities is seen on this patient's ECG? A. 2nd degree heart block - Mobitz type I B. Left atrial enlargement C. Left ventricular hypertrophy D. Right atrial enlargement E. Right ventricular hypertrophy

D. Right atrial enlargement Right atrial enlargement is most commonly due to pulmonary hypertension, brought on by chronic lung disease (resulting in cor pulmonale), tricuspid stenosis, or congenital heart disease such as pulmonary stenosis or tetralogy of Fallot. It is characterized by an increase in P-wave amplitude of >2.5 mm.

A 75-year-old man is brought to the emergency department because of a productive cough and a fever of 39.4°C (103°F) for the past 3 days. The man has dementia and is brought to the hospital from a nursing home. A sputum gram stain shows lancet shaped gram positive diplococci. A chest radiograph shows the following: A. Aspiration pneumonia B. Interstitial Pneumonia C. Small cell lung carcinoma D. Lobar Pneumonia E. Sarcoidosis

D. Lobar Pneumonia Lobar pneumonias show up as localized consolidations on chest radiographs. Streptococcus pneumoniae is a gram positive diplococci and is the most common cause of lobar pneumonia.

A 50-year-old man comes to the office because of two weeks of fevers that he has measured to be as high as 100°F (38.8°C). He has had an occasional dry cough and chest pain during inspiration. Yesterday, there were some streaks of blood in his cough, which prompted him to come in for evaluation. He had not had any rhinorrea or sore throat. He works as a prison guard and smokes half a pack of cigarettes per day. He has a history of diabetes mellitus for which he takes metformin, and was last checked one year ago. A chest x-ray is taken and shown below. Which of the following is the most likely diagnosis? A. Sarcoidosis B. Septic embolus C. Klebsiella pneumoniae infection D. Mycobacterium tuberculosis infection E. Bronchiectasis

D. Mycobacterium tuberculosis infection Mycobacterium tuberculosis infection is declining in the United States. However, this patient's occupation and his diabetes (potentially uncontrolled due to his lack of follow-up) are risk factors. M. tuberculosis is an aerobic, slow-growing bacillus. When contaminated droplets are inhaled, the organism is taken up by alveolar macrophages, but the macrophages are unable to kill and digest the bacterium, leading to caseating granuloma formation

What is the treatment that is most commonly indicated in pneumothorax? A. Rigid bronchoscopy B. Flexible bronchoscopy C. Watchful waiting D. Needle decompression

D. Needle decompression

Which of the following anti-TB drugs require renal dose adjustments? A. INH B. RIF C. INH & RIF D. PZA and EMB

D. PZA and EMB

A 73-year-old woman comes to the emergency department because she sustained multiple stab wounds from a knife to the chest during a mugging 20 minutes ago. She says the pain is worse on inspiration and that she feels shortness in her breath. Her temperature is 36.8°C (98°F), pulse is 104/min, respirations are 26/min, and blood pressure is 95/78 mm Hg. Chest X-ray is obtained and is shown below. Which of the following is the deepest layer of tissue that the offending weapon penetrated? A. Dermis B. Epidermis C. Intercostal muscle D. Parietal pleura E. Pectoral muscle

D. Parietal pleura The patient has a massive left sided tension pneumothorax. In this particular case, the pleural space was penetrated by the knife which allowed air to enter but not to exit. The resulting build up of air has caused pressure to build up in the pleural space resulting in a collapsed lung and mediastinal shift. The buildup of pressure in the chest cavity can collapse the vena cava causing a complete circulatory collapse in minutes. Needle decompression followed by a chest tube is indicated.

A 23-year-old college student comes to the student health offices because of a non-productive cough, subjective tactile fevers, and "feeling sick" for 2 days. She says she has a final exam tomorrow and wants some medicine so she feels better during the test. Her temperature is 37.2°C (99°F), pulse is 87/min, respirations are 18/min, and blood pressure is 117/78 mm Hg. Examination shows clear lung sounds bilaterally in all fields. There is no cervical lymphadenopathy. She has some pharyngeal erythema with neither exudates nor vesicles. Which of the following is the most appropriate next step in the management of this patient? A. Admit for IV antibiotic therapy B. Chest X-ray C. Outpatient antibiotic therapy D. Reassure and discharge E. Respiratory viral panel by PCR

D. Reassure and discharge Upper respiratory tract infections can be diagnosed solely on the basis of history and exam findings and do not require further work-up or treatment. For cases which are less clear, the modified Centor score can be used to triage patients to a combination of further testing and/or empiric antibiotic treatment.

A 67-year-old man is brought to the emergency department by his wife because of shortness of breath for the past 3 hours. The patient has a history of chronic obstructive pulmonary disease not treated with any medications. His vital signs are significant for a heart rate of 115, and an oxygen saturation of 85% on room air. Prior to initiation of supplemental oxygen, an arterial blood gas (ABG) is drawn showing the following results:pH 7.31 PaO2 57 mmHg HCO3- 26 mEq/L PaCO2 52 mmHg Which of the following is most consistent with this patient's ABG data? A. Metabolic acidosis with respiratory alkalosis B. Metabolic alkalosis C. Metabolic acidosis D. Respiratory acidosisE. Respiratory acidosis with metabolic alkalosis

D. Respiratory acidosis When given an arterial blood gas problem, the first step is to look at the patient's pH to determine whether the patient is acidemic or alkalemic. Respiratory acidosis is characterized by elevated levels of PaCO2 and normal or slightly elevated levels of HCO3- .

A 43-year-old woman comes to the emergency department because of palpitations for the past hour. An arterial blood gas is drawn and shows the following results: PaO2 98 mmHg (13.1 kPa) pH 7.64 PaCO2 20 mmHg (2.71 kPa) HCO3- 23 mmol/L Which of the following is most consistent with her results? A. Metabolic alkalosis B. Mixed metabolic and respiratory acidosis C. Metabolic acidosis D. Respiratory alkalosis E. Respiratory acidosis

D. Respiratory alkalosis A hyperventilating patient is likely to have a respiratory alkalosis because of her extremely elevated minute ventilation (PaCO2 is inversely proportional to minute ventilation). In a respiratory alkalosis, the pH will be elevated and the PaCO2 will be markedly decreased.

A 52-year-old woman comes to the office because of increasing shortness of breath and nonproductive cough for five months. Her respiratory rate is 23/min at rest. Chest X-ray shows an increase in intercostal spaces bilaterally and an extensive increase in interstitial markings. Spirometry shows marked restrictive ventilatory pattern. Chest CT scan shows areas of honeycombing mostly in peripheral lung bases and cysts bilayers. She was diagnosed with psoriasis vulgaris ten years ago, for which she is taking methotrexate. Which of the following is the most appropriate next step in diagnosis? A. Fibre optic bronchoscopy B. Lung biopsy C. Methacholine challenge D. Right heart catheterization E. Sweat test

D. Right heart catheterization Right heart catheterization is the best investigation for diagnosing pulmonary hypertension, which may be present in patients treated with methotrexate who have developed pulmonary fibrosis. A mean pulmonary arterial pressure (mPAP) of more than 25 mm Hg is diagnostic of pulmonary hypertension.

A 29-year-old female comes to the office because of a cough and "feeling sick" for 2 days. Her medical history is noncontributory and she says she took some acetaminophen for this illness before coming in. Her temperature is 37.2°C (99°F), pulse is 87/min, respirations are 18/min, and blood pressure is 117/78 mm Hg. Examination shows mild nasal congestion, rhinorrhea, and a non-productive cough. She has some pharyngeal erythema with neither exudates nor vesicles. Which of the following is the most accurate test in the diagnosis of this illness? A. Chest X-ray B. Rapid streptococcus antigen detection C. Sputum bacterial culture D. Sputum viral culture E. Sputum viral panel by PCR

D. Sputum viral culture Upper respiratory tract infections usually have a viral rather than bacterial cause. Symptoms that are indicative of a viral infection include; sore throat, non-productive cough, malaise and low grade fever. A viral sputum culture (light microscopy AFB) is the gold standard to confirm suspected cases. *OFTEN NOT NEEDED IN REAL PRACTICE

Which of the following is the recommended treatment for nonsevere bronchiolitis? A. Antibiotics B. Antibiotics and bronchodilators C. Bronchodilators D. Supportive care

D. Supportive care Bronchiolitis Patient will be an infant Difficulty breathing PE will show respiratory distress, polyphonic wheezing, and rales Diagnosis is made by history and physical exam Most commonly caused by respiratory syncytial virus (RSV) Treatment is supportive care

An 85-year-old woman presents to clinic with an inspiratory wheeze and nonproductive cough. She is a nursing home resident who also suffers from dementia, aphasia, and right-sided weakness secondary to an ischemic stroke. Nursing home staff report the wheeze and cough have been present for two weeks with no apparent precipitating factors. Vital signs, including pulse oximetry, are within normal limits. Physical exam demonstrates stable right-sided weakness, no pedal edema, and inspiratory wheeze. Which of the following is the most likely diagnosis? A. Acute exacerbation of chronic obstructive pulmonary disease B. Acute pulmonary embolus C. Atypical angina D. Tracheal foreign body

D. Tracheal foreign body Tracheal foreign body is most common in children under 15 years of age, and deaths from foreign body aspiration are highest in children under 1 year old and adults over 75 years old. Typical aspirates among children include nuts, seeds, and other organic materials. Foreign body aspirates in adults are more varied, including organic material like food and bones or inorganic material such as nails, pins, and dental debris. Elderly, confused, debilitated patients with a weakened swallow mechanism are especially at risk for foreign body aspiration. Other risk factors for foreign body aspiration in adults include drug and alcohol intoxication, trauma, or anesthesia. Airway aspirations may occur anywhere along the tracheobronchial tree. Smaller particles often lodge in the right mainstem bronchus or smaller bronchioles on the right, because this bronchus has a more perpendicular course and is often wider than the left. Larger inhaled particles can lodge in the trachea. These objects may cause complete blockage of the airway and subsequent death due to asphyxia. If air is able to move around the tracheal foreign body, symptoms can include inspiratory (monophasic) wheeze, nonproductive cough, and chest pain. If the course of the obstruction is prolonged, pneumonia, fever, foul-smelling sputum, or hemoptysis may develop. A patient who presents with acute asphyxia must receive oxygen and be intubated. Direct laryngoscopy is performed to visualize the obstruction. Removal of the foreign body with forceps can then be attempted. In non-life-threatening clinical scenarios, such as the vignette above, chest X-ray or chest computed tomography may be used to visualize a radiopaque airway lesion. If no lesion is visualized but suspicion of foreign body aspiration remains high, direct flexible bronchoscopy should be performed.

On CXR, you see enlarged pulmonary arteries, intersitial edema, and signs of R. sided heart failure. What is your diagnosis?

Pulmonary HTN S/S: -Dyspnea, cyanosis -Fatigue -Chest pain -Edema -Exertional symptoms if severe Accentuated S2: -Due to prominent P2 -May have fixed or paradoxically split P2 Signs of R. sided heart failure: -Inc. JVP -Peripheral edema -Ascites

What is the most common presenting symptom in patients with a pulmonary embolism? Cough Dyspnea Hemoptysis Syncope

Dyspnea Pulmonary Embolism Patient will be complaining of dyspnea (most common symptom) PE will show tachypnea (most common sign) ECG will show sinus tachycardia, nonspecific ST segment and T wave changes, right heart strain, S1Q3T3 (classic finding) CXR will show nonspecific abnormalities, Hampton hump (pleural-based wedge infarct), Westermark sign (vascular cutoff sign) Diagnosis is made radiographically - CT pulmonary angiography most preferred Most commonly originate in the lower extremities and pelvis Treatment is anticoagulation (heparin, LMWH, novel oral anticoagulant (NOAC)), supportive care, thrombolytics in hemodynamically unstable patients Comments: In low clinical suspicion: negative D-dimer excludes PE

A 28-year-old HIV-positive man presents to the clinic with a dry cough, fever, and progressive shortness of breath over the past three weeks. Diffuse pulmonary crackles are auscultated bilaterally on physical exam. Which of the following chest X-ray results would be most consistent with a diagnosis of Pneumocystis jirovecii pneumonia in this patient? Cardiomegaly with bilateral pleural effusions Cavitary lesions of the upper lobes Diffuse interstitial infiltrates Reticular opacities with superimposed consolidation

Diffuse interstitial infiltrates Pneumocystis jirovecii pneumonia is a fungal opportunistic infection transmitted via the airborne route. Patients at increased risk of contracting Pneumocystis jirovecii pneumonia include those with HIV who have high viral RNA counts, those with CD4 counts under 200 cells/microL, those with recent episodes of bacterial pneumonia, those with previous infections with Pneumocystis jirovecii, and those with oral thrush. Common signs and symptoms include fever, fatigue, and dry cough with progressively worsening dyspnea. Diagnosis is made based on the clinical picture and consistent radiographic and laboratory findings. Common radiographic findings include bilateral diffuse interstitial infiltrates or diffuse alveolar infiltrates. Laboratory findings consistent with Pneumocystis jirovecii pneumonia include CD4 count under 200 cells/microL, hypoxia, increased lactate dehydrogenase levels (LDH), and elevated serum levels of 1-3-beta-d-glucan. Treatment of Pneumocystis jirovecii pneumonia is with trimethoprim-sulfamethoxazole. Corticosteroids are also administered to those with moderate or severe disease. Prophylactic antibiotics (trimethoprim-sulfamethoxazole) should be given to HIV-positive patients whose CD4 count falls below 200 cells/microL. Pneumocystis Pneumonia (PCP) Patient with a history of HIV, immunocompromised or immunosuppression Gradual onset of nonproductive cough Labs will show CD4 < 200/mm3, increased LDH CXR will show bilateral infiltrates (batwing pattern) Most commonly caused by Pneumocystis jirovecii Treatment is TMP-SMX

A 50-year-old man comes to the office because of frequent coughing with "cups" of mucus for two months. He says he has experienced similar episodes in the preceding two years, which lasted for 3-5 months each time. Examination shows a BMI of 31, central cyanosis, and coarse rhonchi and wheezing are heard on pulmonary auscultation. Oxygen saturation measured by pulse oximetry is 90%. He has a 30 pack-year history of cigarette smoking and his Reid index was calculated to be 55%. Which of the following is the most likely diagnosis? A. Sarcoidosis B. Asthma C. Congestive heart failure D. Emphysema E. Chronic Bronchitis

E. Chronic Bronchitis Chronic bronchitis is a form of chronic obstructive pulmonary disease (COPD) clinically defined as a productive cough for at least 3 months per year for at least 2 years. On pathologic examination, the Reid index will be greater than 50%.

A 42-year-old man comes to the clinic for evaluation of a productive cough with purulent secretions. His symptoms have been going on for the past three days. He also has been experiencing subjective fevers during this same time period. Past medical history is notable for hypertension, which is well controlled with lisinopril. He has a 15-pack-year smoking history and drinks 2-3 beers on the weekends. Temperature is 38.5°C (101.3°F), pulse is 104/min, respirations are 18/min, and blood pressure is 122/88 mmHg. Physical examination demonstrates dullness to percussion and bronchial breath sounds over the left lung. Chest radiograph reveals a left lower lobe consolidation. Which of the following describes the epidemiologically most likely pathogen responsible for this patient's illness? A. Gram-positive cocci in clusters B. Gram-positive bacilli C. Gram-negative bacilli D. Gram-negative coccobacilli E. Gram-positive diplococci

E. Gram-positive diplococci Community-acquired pneumonia describes an infection of the lung tissue that is acquired outside of a healthcare setting. The most common cause of CAP is Streptococcus pneumoniae, which under microscopy appears as Gram-positive diplococci.

A 46-year-old female presents to the emergency department with complaints of fever, cough, and night sweats over the past 2-3 months. Past medical history is notable for HIV infection, type II diabetes mellitus, and hypertension. She is currently undomiciled, drinks 3-4 alcoholic drinks per day, smokes one pack of cigarettes per day, and uses intravenous heroin daily. Temperature is 37.2 °C (99.0°F), pulse is 87/min, respirations are 16/min, blood pressure is 156/74 mmHg, and O2 saturation is 95% on room air. Physical exam demonstrates a thin, cachectic middle aged female with track marks noted on the upper extremities bilaterally. Cardiopulmonary exam reveals rales in the upper lobe of the left lung, without gallops, rubs, or murmurs. Chest x-ray is demonstrated below. A. Increased lymphatic drainage B. Impaired phagolysosome formation by macrophages C. Embolization of bacterial vegetations D. Inhibition of opsonization E. Higher oxygen tension in upper lung lobes

E. Higher oxygen tension in upper lung lobes Reactivation tuberculosis refers to recrudescent infection with Mycobacterium tuberculosis after a period of dormancy. The condition preferentially infects the upper lobes of the lung due to higher oxygen tension and poor lymphatic drainage.

A 45-year-old male with a history of HIV presents to the primary care physician for worsening cough, night sweats, and chills. He has been non-compliant with HAART therapy and smokes one pack of cigarettes daily. His most recent CD4 count is less than 50, and he is empirically initiated on a medication that works by inhibiting the enzyme arabinosyl transferase. Which of the following interventions should be implemented after prescribing the aforementioned medication? A. Supplemental pyridoxine B. Periodic auditory screening C. Obtaining regular liver function tests D. ECG screening E. Periodic visual acuity testing

E. Periodic visual acuity testing This patient with a history of HIV and CD4 count less than 50 presents with symptoms concerning for Mycobacterium avium complex infection. He is started on ethambutol, which works by inhibiting the enzyme arabinosyl transferase. Patients taking ethambutol should undergo period visual acuity assessments to screen for signs of optic neuritis.

A 66-year-old man comes to the emergency department because of a 2-day history of a productive cough with yellow sputum, chest tightness, fatigue, chills, and fever. He has smoked cigarettes since the age of 25. Vital signs show his temperature is 38.8°C (102°F), pulse is 80/min, respirations are 22/min, and blood pressure is 140/90 mm Hg. Auscultation of his lungs shows rhonchi over the lower 2/3 of the left lung and bilateral crackles at the bases. The patient is told to pronounce the long-E vowel sound while you auscultate the lungs, the E sounds like an A over the L lung. Tactile fremitus is increased in this area and the percussion note is dull. Which of the following is the most likely diagnosis? A. Lung cancer B. Atelectasis C. Tuberculosis D. Bronchitis E. Pneumonia

E. Pneumonia Common symptoms of pneumonia include: cough, fevers, chills, fatigues, rigors and pleuritic pain. The diagnosis can often be made on history and examination alone. Key historical factors to explore include: recent respiratory illness, travel, smoking and occupational risks.

A 14-year-old girl with hereditary spherocytosis comes to the emergency department following a fall from a horse. Her injuries consisted of three fractured ribs, a punctured right lung, and a ruptured spleen that required surgical removal. Three months later the patient comes to the office because of fatigue, cough, and shortness of breath. Physical examination shows dullness to percussion and decreased breath sounds in the right lower lobe. A chest radiograph is obtained and shows a right lower lobe infiltrate with patchy opacity. The patient is started on antibiotic therapy. Which of the following organisms is the most likely causative organism in this patient? A. Klebsiella pneumoniae B. Neisseria meningitidis C. Escherichia coli D. Staphylococcus aureus E. Streptococcus pneumoniae

E. Streptococcus pneumoniae Streptococcus pneumoniae is the most common cause of community acquired pneumonia and is also an encapsulated organism. Immunization is part of the childhood immunization schedule for all children but for patients who are asplenic an additional dose of the vaccine is required.

A 13-year old boy is brought to the emergency department because of a fever and a productive cough for the last 24 hours. The patient states that he is having difficulty breathing and is short of breath even without exertion. His mother noticed that he has a poor appetite, has been low in energy, and has missed several soccer practices this week. His mother also mentions that several students in his classroom are also experiencing the same symptoms. Physical examination of the patient shows dullness to percussion in the lower lobes and crackles on chest auscultation. Laboratory analysis of a sputum sample shows gram-positive catalase-negative diplococci. Which of the following is the most likely microorganism causing the patient's symptoms? A. Chlamydia trachomatis B. Haemophilus influenzae C. Mycobacterium tuberculosis D. Staphylococcus aureus E. Streptococcus pneumoniae

E. Streptococcus pneumoniae Streptococcus pneumoniae is a gram-positive catalase-negative diplococcus and the most common cause of community-acquired pneumonia in children. It can also cause otitis media, meningitis, bacteremia, and sinusitis.

A 36-year-old woman comes to the emergency department because of coughing, chest pain, severe shortness of breath, and fever for 9 days. She states that this is the sickest she has felt in a long time. Physical examination shows decreased lung sounds on the right side. Pneumonia is suspected, and a CT scan is obtained. A thoracentesis is also attempted, but does not produce significant drainage. Which of the following is the most appropriate next step? A. Chemical pleurodesis B. Culture of thoracentesis samples, followed by tailoring of antibiotic treatment C. Placement of further chest tubes D. Pneumonectomy E. Surgical decortication

E. Surgical decortication Pleural empyemas, which appear as dense pleural effusions on a CT scan, can develop thick rings that inhibit drainage via thoracentesis/chest tube. In these cases, surgical decortication is required.

A 68-year-old man comes to the emergency department with difficulty breathing, high fever, and intermittent diarrhea. Several other elderly individuals have been reported being admitted to other hospitals in the same area. All patients recently shopped at the local grocery store, where they purchased fresh vegetables stored under the water mist machines. There is some difficulty in obtaining sputum samples. Which of the following samples should be obtained in order to test for the suspected causative organism? Elimination tool A. Blood B. Stool C. Throat swab D. Tissue biopsy E. Urine sample

E. Urine sample Legionnaire's disease is an infection caused by Legionella pneumophila. Antigens can be detected in urine samples. Patients should be screened with both a urine antigen test as well as attempted sputum culture on buffered charcoal yeast extract agar.

A 67-year-old man presents to the office with worsening orthopnea, dyspnea on exertion, and paroxysmal nocturnal dyspnea. He has a history of left-sided systolic heart failure. Exam reveals a new pulmonic murmur. You suspect pulmonary hypertension. What is the best choice for your initial diagnostic study? Chest X-ray Echocardiogram Electrocardiogram Right heart catheterization

Echocardiogram Pulmonary hypertension is a disease of increased pulmonary artery pressure and defined as mean pulmonary arterial pressure greater than 25 mm Hg at rest. There are multiple etiologies which are split into classes. Group 1 is pulmonary artery hypertension (PAH), group 2 is pulmonary hypertension due to left heart disease, group 3 is pulmonary hypertension due to chronic lung disease or hypoxemia, group 4 is chronic thromboembolic pulmonary artery hypertension, and group 5 is pulmonary hypertension due to unclear multifactorial mechanisms. A patient with known left heart systolic failure is suspected to be in group 2. Initial symptoms are a result of the inability to increase cardiac output during exercise. The patient will develop exertional dyspnea and fatigue. Symptoms progress as the pulmonary hypertension advances to include angina, exertional syncope, peripheral edema, and anorexia from abdominal ascites associated with right-sided heart failure. Initial exam findings are an loud pulmonic component of the second heart sound (P2) or the second heart sound (S2) may be narrowly split if right ventricular function is maintained. As the right ventricular pressure increases, an A wave jugular venous pulse may develop a fourth heart sound or a heave along the parasternal border. In more severe disease, as the right ventricle fails and develops a right bundle branch block, there may be a tricuspid murmur. In even further advanced disease, a pulmonic regurgitation murmur may develop. The goal of diagnostic studies when pulmonary hypertension is suspected is to confirm diagnosis and determine the etiology. The first step in diagnosis is echocardiogram. The echocardiogram should be interpreted to determine if the left heart failure is significant enough to increase pulmonary artery pressures and create pulmonary hypertension. This also assesses the function and extent of valvular dysfunction. If the echocardiogram shows enough left heart failure to explain the pulmonary hypertension, then no further workup is needed. If there is not a clear justification for the pulmonary hypertension on echocardiogram, then further testing, such as pulmonary function testing, ventilation-perfusion scanning, and overnight oximetry, should be performed. Right heart catheterization (definitive diagnosis) is indicated in patients who still are not found to have a clear explanation of the pulmonary hypertension. Once the etiology is determined, then management is directed at the underlying etiology. Vasodilators may lower pulmonary vascular resistance in some patients. Available vasoactive agents include oral CCBs (first line for primary pulmonary HTN), inhaled phosphodiesterase inhibitors, prostacyclins, and endothelin receptor antagonists.

Which of the following is a risk factor for pulmonary infection with Histoplasmosis capsulatum? Changing cat litter while pregnant Exploring caves in the Ohio River Valley Farming in Arizona Tending pigeons in the Pacific Northwest

Exploring caves in the Ohio River Valley Histoplasmosis Patient with a history of travel to Ohio or Mississippi River valley and exposure to bird or bat droppings X-ray will show pulmonary infiltrates (lobar or diffuse reticulonodular), hilar and mediastinal adenopathy Diagnosis is confirmed by culture Treatment is itraconazole or amphotericin B

____________ can be given to immunocompromised patients to prophylactically suppress cryptococcal meningitis.

Fluconazole

Hallmark plexiform lesions, classified as glomeruloid vascular lesions that form at arterial branches, are seen in _________________.

Group 1 PAH

A previously healthy 28-year-old woman presents to your clinic with a complaint of a productive cough for six days. She says that her symptoms started with nasal congestion, headache, and sore throat for about two days, then the cough started. She has not had a fever and is diagnosed with acute bronchitis. Which of the following is the most appropriate therapy? Azithromycin Guaifenesin Influenza vaccination Prednisone

Guaifenesin Acute bronchitis is one of the most common medical conditions for which people seek treatment. It occurs when there is inflammation of the bronchi, air passages, small airways, and alveoli of the lungs. The most common cause is a viral infection. Patients present with cough for at least five days, with or without sputum, and may describe symptoms of an upper respiratory illness just prior to the cough starting.

Which of the following is an absolute contraindication to fibrinolytic therapy in a patient with a pulmonary embolism? History of a hemorrhagic stroke Menstruation Surgery within the preceding 10 days Systolic blood pressure above 200 mm Hg

History of a hemorrhagic stroke absolute or major contraindications generally include an intracranial neoplasm, recent (within the past two months) intracranial or spinal surgery or trauma, history of a hemorrhagic stroke, active bleeding or bleeding diathesis, or nonhemorrhagic stroke within the previous three months. Relative contraindications include severe uncontrolled hypertension (systolic blood pressure above 200 mm Hg or diastolic blood pressure above 110 mm Hg), nonhemorrhagic stroke older than three months, surgery within the previous 10 days, and pregnancy. Alternative therapies in patients with contraindications to thrombolytic therapy who are hemodynamically unstable include catheter and surgical embolectomy

On CXR, emphysema will present as (hyperlucent/hypolucent) _____________ along with hyperexpanded lungs, flattened diaphragms, and decreased lung markings.

Hyperlucent

Airway obstruction, chronic lung disease, opioids, sedatives, and weakening of respiratory muscles cause ______________ and therefore respiratory acidosis.

Hypoventilation

In group 1 PHTN (PAH) you will see an (decrease/increase) __________ in endothelin levels, (decrease/increase) ____________ in nitric oxide levels, and a (decrease/increase) __________ prostacyclin levels.

Increase (endothelin is a vasoconstrictor and mitogen) Decrease (nitric oxide is a vasodilator and is antiproliferative) Decrease (prostacyclin is a vasodilator, and is an antiproliferative, and inhibits platelet function)

The initial disturbance of respiratory acidosis is an increased Pco2 concentration, followed by a compensatory response of (increased/decreased) ______________ HCO3-.

Increased

The (Mantoux TST/Interferon Gamma Release assay) _____________ is able to distinguish between latent and active TB.

Inferferon Gamma Release Assay - blood test

A team of researchers is attempting to develop a new pharmacotherapy for the treatment of respiratory infections. Pathogen X, as identified by the researchers, is introduced to an in vitro model consisting of human nasopharyngeal epithelial cells grown within a culture container. Pathogen X is found to produce an enzyme that binds sialic acid residues on the surface of nasopharyngeal epithelial cells. This enzyme allows pathogen X to subsequently enter the epithelial cells. Clinically, pathogen X causes symptoms including shortness of breath, fevers, myalgias, and joint pain. Furthermore, infection by pathogen X can predispose patients to subsequent infection by Staphylococcus aureus. Which of the following is the most likely identity of pathogen X? A. Influenza virus B. Mycoplasma pneumoniae C. Streptococcus pneumoniae D. Cytomegalovirus E. Legionella pneumophila

Influenza virus

________________ is the most common cause of atypical pneumonia in the elderly/immunocompromised.

Influenza virus

A 70-year-old man with chronic obstructive pulmonary disease presents to the office concerned about getting sick this winter. He states that a friend of his suffered severely from the flu and was admitted to the ICU for a couple of weeks. He is up-to-date on his childhood vaccinations. Which of the following vaccinations can help reduce his risk for infections and exacerbations? Influenza and pneumococcal polysaccharide vaccinations Influenza vaccination Influenza, measles, mumps, and rubella vaccinations Influenza, pneumococcal polysaccharide, and pneumococcal conjugate vaccinations

Influenza, pneumococcal polysaccharide, and pneumococcal conjugate vaccinations

The ______________ phase of TB treatment is characterized by 4 drugs (RIPE) being administered x 2 months, while the _____________ phase of TB treatment is characterized by 2 drugs (RI) being administered x 4 (or more) additional months.

Initial intensive phase...... Continuation phase

A 34-year-old man presents with cavitations on chest radiograph and three positive acid-fast bacillus sputum stains. He was recently in prison and states that his cellmate had active tuberculosis. The patient is HIV-negative. Assuming adequate treatment response and bacterial susceptibility, which of the following treatment regimens is recommended for this patient? Isoniazid, rifampin, ethambutol, and pyrazinamide for nine months Isoniazid, rifampin, ethambutol, and pyrazinamide for two months, followed by four months of isoniazid and rifampin Isoniazid, rifampin, streptomycin, and pyrazinamide for four months, followed by five months of isoniazid Isoniazid, rifampin, streptomycin, and pyrazinamide for six months

Isoniazid, rifampin, ethambutol, and pyrazinamide for two months, followed by four months of isoniazid and rifampin

(Kidney/respiratory) _____________ compensation is the most effective regulator of pH.

Kidney excretion

The (killed/attenuated) ______________ influenza vaccine is appropriate for immunocompromised individuals.

Killed

Caseating granuloma is associated with the (primary/latent/reactivation) _____________ phase of TB.

Latent phase -On smear, able to see Langerhan's Giant Cells

Mycoplasma, _______________ and Chlamydia are bacteria associated with atypical pneumonia.

Legionella

A 58-year-old man presents to the emergency department complaining of shortness of breath, cough, fever, nausea, and diarrhea for three days. He recently installed a hot tub in his home and has been using it quite frequently. Physical exam reveals a soft, mildly tender abdomen, bilateral crackles on chest auscultation, and a pulse oxygen saturation of 91%. Abnormal laboratory values include leukocytosis, hyponatremia, and elevated liver enzymes. Which of the following causes of atypical pneumonia correlates most closely with this patient's clinical picture? Klebsiella pneumoniae Legionella pneumoniae Mycoplasma pneumoniae Pseudomonas aeruginosa

Legionella pneumoniae is a gram-negative, aerobic organism responsible for both nosocomial and community-acquired pneumonia. The bacteria are present in water and soil, and infection occurs via contaminated aerosolized particles. Although the bacteria are ubiquitous in the environment, most human infections occur as a result of contaminated water from man-made sources, such as pools, hot tubs, air conditioning units, and drinking water. The presenting signs and symptoms of pneumonia caused by Legionella include respiratory as well as gastrointestinal disturbances. Treatment of Legionella pneumoniae is with a macrolide, a respiratory fluoroquinolone, or doxycycline, although the organism is often resistant to the tetracyclines. Prevention of the infection is through purification of the water supply and good handwashing after working in soil.

A 60-year-old man presents to the emergency department with cough and shortness of breath for the past two days. He lives in an old apartment building. Vital signs are T 101.6°F, HR 85, BP 126/76 mm Hg, RR 16, and oxygen saturation is 98% on room air. Complete blood count and chemistry panel reveal leukocytosis and hyponatremia. Chest X-ray shows a unilobar infiltrate. Microbiologic testing is pending. Which of the following is the most likely diagnosis? Acute bronchitis Chronic obstructive pulmonary disease Legionnaires disease Lung cancer

Legionnaires disease Legionnaires disease (Legionella pneumonia) is the most common clinical manifestation of Legionella infection. The symptoms typically begin two to 10 days after exposure to contaminated water or soil. Risk factors include older age, smoking, chronic renal or pulmonary disease, and immunocompromised status. Pneumonia caused by Legionella species has similar clinical and radiograph findings to other causes of pneumonia. The classic symptoms are fever, cough, and shortness of breath. Gastrointestinal symptoms, such as nausea, vomiting, and diarrhea, are also common. Findings on physical exam may include signs of consolidation, such as rales, tactile fremitus, decreased breath sounds, and bronchophony. Large outbreaks are often associated with contaminated water systems that supply communities such as hospitals, hotels, cruise ships, and apartment buildings.

If it becomes a severe, systemic mycosis, blastomycosis may be treated with ________________.

Liposomal Amphotericin B *Mild-moderate disease treated with oral Itraconazole

A 19-year-old woman from Wisconsin with no significant past medical history presents to urgent care for a five-day history of fever, cough, and myalgias. When asked about her social history, she reports that she frequently participates in a river clean-up project with her local college, she likes to collect houseplants, she visited family in Arizona a few weeks ago, and she also volunteers at a sheep farm. What part of her history makes histoplasmosis most likely? Close proximity to sheep Collecting houseplants Living in Wisconsin Traveling to Arizona

Living in Wisconsin Histoplasmosis is a fungal infection that is commonly found in bird and bat droppings along the Ohio River and Mississippi River Valleys, including Wisconsin, Minnesota, Iowa, Illinois, Michigan, Ohio, Indiana, and Kentucky. The most high-risk areas are caves, river beds, and abandoned buildings. Patients become infected when they inhale the fungal spores, which then proliferate in the lungs. Most patients are asymptomatic, however, some will report fever and a nonproductive cough that lasts 5 to 10 days beginning several weeks after exposure. The disease may become disseminated if the patient is immunocompromised (CD4 cell count < 100 cells/mcL)

A 78-year-old woman with a previous history of left-sided stroke and dysphagia presents to the clinic complaining of weeks of productive cough, fever, malodorous sputum, and weight loss. Chest X-ray reveals a solitary pulmonary infiltrate in the right lower lung with a central cavitation and a visible air-fluid level. Which of the following represents the most likely diagnosis? Aspirated foreign body Lung abscess Lung cancer Sarcoidosis

Lung abscess Lung Abscess Patient will be complaining of several weeks of cough, fever, pleuritic chest pain, weight loss, and night sweats CXR will show area of dense consolidation with an air-fluid level inside a thick-walled cavitary lesion Most commonly caused by aspiration pneumonia Treatment is ampicillin-sulbactam, carbapenems, clindamycin

The influenza virus surface protein ______________ promotes progeny virion release from infected cells.

Neuraminidase

A 16-year-old boy presents to the office with a two-week history of runny nose, sore throat, and nasal congestion. He developed a productive cough with yellowish sputum about one week ago that has not improved. He complains of some fatigue but no body aches or malaise. A rapid influenza test is negative. Upon physical exam, he has a fever of 101.0°F, and there are mild diffuse wheezes throughout the lungs posteriorly on auscultation. Which of the following is the most appropriate next step in management? Obtain a chest radiograph Prescribe a course of clarithromycin for seven days Prescribe a course of oseltamivir for five days Recommend rest, fluids, and symptomatic treatment with antitussives and cough suppressants

Obtain a chest radiograph Acute bronchitis is a very common infection involving an acute inflammation of the bronchi. It can be caused by both bacterial and viral infections, but the offending agent is rarely identified. Patients may present with cough (the most common symptom), sputum production, sore throat, nasal discharge, nasal congestion, headache, and muscle aches. In patients with severe cases of bronchitis, general malaise and chest pain may be present. Dyspnea and cyanosis are observed in patients with underlying lung disease such as chronic obstructive pulmonary disease. Fever is an unusual symptom in acute bronchitis, and if it is seen in conjunction with cough, it is suggestive of pneumonia or influenza.

A PCT elevation may suggest ___________ infection while a low PCT may suggest ___________ infection.

PCT elevation: suggests bacterial infection Low PCT: suggests viral infection OR resolution of bacterial infection

All 5 groups of PHTN require a Ppa > 20mmHg, however, group 1 PAH requires what additional criteria in order to be diagnosed?

PVR > 3 Wood's units (normal 1-2) -Pulmonary capillary wedge pressure < or equal to 15mmHg (reflecting absence of L heart failure) -Absence of other cause for PHTN *Right heart catherization is required for definitive dx of PAH

Cryptococcus neoformans is found in soil and __________ droppings.

Pigeon

A 24-year-old man presents with left-sided chest pain and shortness of breath after being involved in a motor vehicle collision. Lung sounds are decreased on the left side. Chest radiograph is shown above. Which of the following is the most likely diagnosis? Hemothorax Pericardial effusion Pneumothorax Pulmonary contusion

Pneumothorax

A 22-year-old man presents to the emergency department with a sudden-onset of dyspnea and pleuritic chest pain. Symptoms are present at rest. The patient is a smoker, tall and thin, but has no known lung disease or other chronic illness. His breath sounds are diminished unilaterally. His chest X-ray is shown above. Which of the following is the most likely diagnosis? Costochondritis Pleurisy Pneumonia Pneumothorax

Pneumothorax Spontaneous Pneumothorax Patient will be a young, tall, thin, man PE will show decreased breath sounds, decreased fremitus, hyperresonance to percussion CXR will show the absence of lung markings along lung periphery Treatment is: < 20% in a healthy patient: observation with oxygen administration > 20%: chest tube thoracostomy

A 73-year-old woman is in the hospital with exacerbation of her systolic heart failure. 72 hours after admission, she begins to develop a productive cough and shortness of breath. You diagnose her with pneumonia and order sputum cultures. When choosing an antibiotic regimen, which bacteria do you need to ensure is covered until cultures are returned? Clostridium difficile Pseudomonas aeruginosa Streptococcus agalactiae Streptococcus pneumoniae

Pseudomonas aeruginosa Bacterial Pneumonia HAP: starts ≥ 48 hrs after admission VAP: ≥ 48 hrs after endotracheal intubation Pathogens Streptococcus pneumoniae: most common, rust-colored sputum, rigors, gram-positive paired lancets Klebsiella: chronic alcohol use, currant-jelly sputum, bulging fissures Staphylococcus aureus: intravenous drug use, postinfluenza, older adults, gram-positive cocci in clusters Haemophilus influenzae: COPD, gram-negative pleomorphic rods Pseudomonas: cystic fibrosis, nursing home residents, cyanosis

A 32-year-old woman with complaints of sudden onset dyspnea, chest pain, and hemoptysis presents to the emergency department. Her past medical history includes an open reduction with internal fixation of a distal tibia fracture 21 days ago. Which of the following is the most likely diagnosis? Acute myocardial infarction Pneumonia Pulmonary embolism Tuberculosis

Pulmonary embolism Pulmonary Embolism Patient will be complaining of dyspnea (most common symptom) PE will show tachypnea (most common sign) ECG will show sinus tachycardia, nonspecific ST segment and T wave changes, right heart strain, S1Q3T3 (classic finding) CXR will show nonspecific abnormalities, Hampton hump (pleural-based wedge infarct), Westermark sign (vascular cutoff sign) Diagnosis is made radiographically - CT pulmonary angiography most preferred Most commonly originate in the lower extremities and pelvis Treatment is anticoagulation (heparin, LMWH, novel oral anticoagulant (NOAC)), supportive care, thrombolytics in hemodynamically unstable patients Comments: In low clinical suspicion: negative D-dimer excludes PE

A 76-year-old man presents with exertional dyspnea, fatigue, and a cough. History reveals a 45 pack-year smoking history and resultant COPD. An echocardiogram reveals right ventricular wall thickening with paradoxical motion of the interventricular septum during systole. You make the diagnosis of cor pulmonale. Which of the following is the most likely underlying cause? Coxsackievirus Hypertrophic cardiomyopathy Pulmonary hypertension Systemic hypertension

Pulmonary hypertension Cor pulmonale is defined as a structural and functional alteration of the right ventricle due to an underlying lung disease, such as pulmonary hypertension resulting from COPD. The most common cause of chronic cor pulmonale is COPD and the most common cause of acute cor pulmonale is pulmonary embolism. The symptoms of cor pulmonale may be very subtle but may include exertional dyspnea, fatigue, cough, tachypnea, and anginal chest pain. Physical exam may reveal an increased chest diameter, labored breathing with retractions, cyanosis, digital clubbing and wheezes or crackles on lung auscultation may be heard, which reflect the underlying lung disease. Signs of right ventricular failure such as hepatomegaly, edema, JVD may also be noted. Several tests are used to diagnose cor pulmonale, including routine lab work, chest radiography, ECG, echocardiography, and right heart catheterization. CXR may reveal enlargement of the RA, RV, and pulmonary arteries. ECG may show right axis deviation, P pulmonale, and RVH. Echocardiogram may reveal elevated pulmonary artery systolic pressures, and altered right-sided ventricle structure or dysfunction. Right heart catheterization is the gold standard test to confirm the diagnosis of pulmonary hypertension. This test is invasive, so not every patient requires this procedure. Once the diagnosis of cor pulmonale is made, the underlying lung pathology must be diagnosed and addressed as well. Treatment of cor pulmonale is to treat the underlying lung condition.

A 71-year-old woman presents to the clinic for dyspnea on exertion and pedal edema. She has a 25 pack-year smoking history. On physical exam, a loud second heart sound is noted, with a widely split S2, S4, and a holosystolic murmur that is loudest during inspiration at the third intercostal space to the left of the sternum. ECG reveals right bundle branch block. Which of the following diagnoses is most likely? Aortic valve stenosis with left heart failure Myocardial infarction with left heart failure Pulmonary hypertension with right heart failure Pulmonic valve stenosis with right heart failure

Pulmonary hypertension with right heart failure Pulmonary Hypertension Mean PA pressure of 20 mm Hg or more Patient complaining of exertional dyspnea, syncope, and/or fatigue Physical exam: Initially: often cyanosis, usually increased intensity of the second heart sound As PH progresses - signs of RV failure JVP is typically elevated, prominent a wave, prominent v wave (seen often in severe RV failure) A right-sided third or fourth heart sound (ie, a gallop) CXR: tapering PAs, RVH Echocardiography is often diagnostic (enlarged RV and RA) Management includes: O2 sat is greater than 90 percent, refer to pulm/cardio specialist Types: Group 1: PAH Group 2: Left heart disease Group 3: Lung Disease Group 4: Chronic thromboembolic pulmonary HTN Group 5: Multifactorial/Unknown

Type A influenza viruses undergo (slow/rapid) _________ genetic changes.

Rapid

A 20-year-old patient presents to the clinic with a chronic cough and loss of appetite. The patient immigrated to the United States four years ago from Nicaragua. Chest X-ray reveals three small cavitary lesions in the apex of the left lung. Purified protein derivative testing is positive, and sputum is positive for acid-fast bacillus. There is no regional lymph node involvement and no evidence of disseminated disease. Which of the following pathologies is most likely? Active primary tuberculosis Latent tuberculosis Nontuberculous cavitary lung disease Reactivation of latent tuberculosis

Reactivation of latent tuberculosis Reactivation of latent tuberculosis usually results in pulmonary cavitary lesions in the apices of the lungs, with less caseation, less regional lymph node involvement, and less disseminated disease than is seen in primary active tuberculosis. Intradermal purified protein derivative is used as a screening test for the disease, and definitive diagnosis is made through positive acid-fast bacillus sputum stains. Treatment of active pulmonary tuberculosis in a patient with susceptible bacteria who is not human immunodeficiency virus-positive involves two months of isoniazid, rifampin, ethambutol, and pyrazinamide, followed by four months of isoniazid and rifampin.

If a (compressive/resorption) _____________ atelectasis, the trachea shifts TOWARD the affected side.

Resorption

55 year-old man with COPD pH 7.25 PCO2 - 70 HCO3 - 26 What is his acid-base status? Respiratory acidosis Metabolic acidosis Respiratory alkalosis Metabolic alkalosis

Respiratory acidosis

A 38 year-old man comes in to the ED after being involved in a rollover motor vehicle crash. He is complaining of R sided CP and difficulty breathing. He is taking shallow breaths. He has symmetric breath sounds. What acid-base disorder do you suspect? Metabolic acidosis Metabolic alkalosis Respiratory alkalosis Respiratory Acidosis

Respiratory acidosis *Shallow respirations (splinting) so build-up of CO2

55 year-old man with COPD pH 7.32 PCO2 - 70 HCO3 - 35 What is his acid-base status now? Respiratory acidosis with compensatory metabolic alkalosis Respiratory alkalosis with compensatory metabolic acidosis Metabolic acidosis with compensatory respiratory alkalosis Metabolic alkalosis with compensatory respiratory acidosis

Respiratory acidosis with compensatory metabolic alkalosis

A 25 year-old man comes in to the office for a refill of his albuterol because he has run out and feels his chest has been tight and is causing him to "breathe more". Metabolic Acidosis Metabolic Alkalosis Respiratory Alkalosis Respiratory Acidosis

Respiratory alkalosis

62 year-old woman with pneumonia for 1 week pH 7.46 PCO2 - 20 HCO3 - 14 What is the acid-base disorder? Respiratory acidosis Metabolic acidosis Respiratory alkalosis Metabolic alkalosis

Respiratory alkalosis *If it was a metabolic alkalosis, the bicarbonate would increase

Which of the following medications used to treat pulmonary tuberculosis can cause orange discoloration of urine? Ethambutol Isoniazid Rifampin Streptomycin

Rifampin Treatment of active infection initially consists of taking rifampin, isoniazid, ethambutol, and pyrazinamide for two months, which is followed by taking rifampin and isoniazid for an additional four months. Each of these four medications can be associated with elevated liver enzymes. Rifampin is known to cause orange discoloration of the urine and thrombocytopenia.

Your patient presents to the ED with foreign body aspiration of a peanut and is in moderate respiratory distress. You expect this FB to be located in which lung?

Right lung (60% of the time)

Which of the following best describes cor pulmonale? Right atrial dilation secondary to chronic lung disease Right atrial hypertrophy secondary to congenital heart disease Right ventricular dilation secondary to left ventricular dysfunction Right ventricular hypertrophy secondary to pulmonary hypertension

Right ventricular hypertrophy secondary to pulmonary hypertension Cor pulmonale describes right ventricular hypertrophy or dilation resulting from pulmonary hypertension. This altered right ventricular structure may be accompanied by right heart dysfunction. Cor pulmonale is the end result of untreated, longstanding pulmonary hypertension in patients with chronic lung disease or chronic hypoxemia. The normal physiologic response to hypoxemia is pulmonary vasoconstriction, and in chronic hypoxemia, the pulmonary vasculature begins to remodel, becoming more fibrotic and thickened. This leads to increased pulmonary vascular resistance and pulmonary hypertension. With prolonged pulmonary hypertension, the right ventricle of the heart becomes enlarged, either hypertrophied or dilated, from contracting against increased pulmonary arterial resistance. Risk factors for the development of pulmonary hypertension include cigarette smoking, obesity, sleep apnea, interstitial lung disease, and living at high altitude. Signs and symptoms of cor pulmonale include dyspnea, lethargy, fatigue, syncope, presyncope, exertional chest pain, palpable pulmonic component of the second heart sound, widely split S2, elevated jugular venous pressure, lower extremity edema, and right axis deviation on electrocardiography.

Your patient presents to the ED and you determine they have obstruction of the proximal broncus by a foreign body through a CXR. What do you do next?

Rigid bronchoscopy under general anesthesia in the OR -Visualizes only the trachea and proximal bronchi and is most commonly used for removal of foreign bodies and interventional stents.

(Rigid/flexible) ___________ bronchoscopy is traditionally preferred in foreign body aspiration.

Rigid traditionally preferred Should evaluate with flexible after removal, or if there are multiple FB or fragments expected.

A 25-year-old African-American woman presents to the clinic with fever, malaise, cough, and dyspnea on exertion for the past two weeks. She also complains of arthralgias. Chest radiograph reveals bilateral hilar adenopathy and pulmonary infiltrates. Biopsy reveals noncaseating granulomas. Mycobacterial and fungal cultures are negative and the patient has no suspicious environmental exposures. Which of the following is the most likely diagnosis? Histoplasmosis Hypersensitivity pneumonitis Sarcoidosis Tuberculosis

Sarcoidosis Sarcoidosis Labs will show hypercalcemia and elevated serum ACE CXR will show bilateral hilar adenopathy Biopsy will show noncaseating granulomas Treatment is steroids Comments: Lupus pernio (chronic, violaceous, raised plaques and nodules commonly found on the cheeks, nose, and around the eyes) is pathognomonic for sarcoidosis and is the most specific physical exam finding in this disease

______________ pulmonary hypertension is caused by hypoxemia, from chronic obstructive pulmonary disease, interstitial lung disease, or increased vascular pressures from congestive heart failure.

Secondary

Reassortment of influenza virus segments is called antigenic (shift/drift) ____________, and causes pandemics.

Shift

Nosocomial pneumonia is most commonly caused by __________________, Pseudomonas, and other enteric gram-negative rods.

Staphylococcus aureus

T or F: For analyzing pH, venous blood is sufficient.

TRUE

A tall, thin, 22-year-old man presents to the emergency department complaining of shortness of breath and chest pain. He reports the symptoms began one hour ago while he was watching TV. He is a nonsmoker and reports having three beers per week. Which aspect of this patient's history is most specific for primary spontaneous pneumothorax? Chest pain Nonsmoking man Shortness of breath Tall, thin, 22-year-old man

Tall, thin, 22-year-old man Unstable patients should undergo chest tube thoracostomy or needle decompression of the pleural space. In instances where chest tube thoracostomy is delayed, needle decompression is performed first. For stable patients with a small primary pneumothorax, treatment with supplemental oxygen and observation is appropriate. For patients with a secondary pneumothorax, chest tube thoracostomy should be considered to minimize the risk of aspiration or prolonged air leak. Patients should be hospitalized and closely observed for risk of respiratory failure, cardiovascular collapse, or recurrent event.

Your patient calls the clinic regarding nasal discomfort, sore throat, and mild hemoptysis that has been present since their bronchoscopy 24 hours ago. What do you tell them?

These findings are not unusual after broncscopy, monitor symptoms but they should resolve.

When does a V/Q mismatch occur?

This happens when part of the lung resceives oxygen without blood flow or blood flow without oxygen.

A(n) ________________ is a type of pleural effusion that has decreased protein content and is often due to congestive heart failure, nephrotic syndrome or hepatic cirrhosis.

Transudative pleural effusion

A 38-year-old man presents with a gradual onset of a cough, fever, and difficulty breathing for three weeks. Past medical history is significant for human immunodeficiency virus infection with a CD4 count of 150 cells/microL. On physical exam, the patient is noted to be tachypneic and febrile. Lung auscultation reveals crackles and rhonchi throughout the lung field. Which of the following treatments is the best therapy? Azithromycin Ceftriaxone Trimethoprim-dapsone Trimethoprim-sulfamethoxazole

Trimethoprim-sulfamethoxazole Pneumocystis Pneumonia (PCP) Patient with a history of HIV, immunocompromised or immunosuppression Gradual onset of nonproductive cough Labs will show CD4 < 200/mm3, increased LDH CXR will show bilateral infiltrates (batwing pattern) Most commonly caused by Pneumocystis jirovecii Treatment is TMP-SMX

A 43-year-old man reports to the emergency department after falling off a ladder at work. He has dyspnea and chest pain with decreased breath sounds on the left compared bilaterally. Plain radiographs of the chest reveal increased air space with sharply demarcated borders and decreased pulmonary markings, which are consistent with a moderate traumatic pneumothorax. What treatment is indicated for this patient's condition? Emergent thoracotomy Expectant management Needle thoracostomy Tube thoracostomy

Tube thoracostomy Emergent thoracotomy (A) is not first-line treatment for patients with isolated traumatic pneumothorax. This treatment is best utilized for those with cardiac tamponade or who quickly destabilize in the emergency department. Expectant management (B) may be warranted for small spontaneous pneumothorax or asymptomatic pneumothorax. This strategy is usually coupled with supplemental oxygen to facilitate resorption of pleural air. Needle thoracostomy (C) is the first-line treatment for a tension pneumothorax, which is indicated on X-ray as a mediastinal shift toward the contralateral pleural space.

Which of the following tests is most commonly used to screen for tuberculosis in asymptomatic individuals? Chest CT Chest X-ray Sputum culture Tuberculin skin test

Tuberculin skin test There are various treatment regimens for latent tuberculosis infection. Rifampin can be taken daily for four months. Another treatment option is taking isoniazid daily for nine months.

A 30-year-old man presents to the clinic during the winter with fever, cough, malaise, headache, and myalgias. He is diagnosed with influenza A. Which comorbidity causes a patient with influenza to be at high-risk for complications? Body mass index of 30 Hyperlipidemia Hypertension Uncontrolled diabetes mellitus

Uncontrolled diabetes mellitus

(Upper respiratory/lower respiratory) _________ infection may reveal conjunctival injection, nasal mucosal swelling, nasal congestion, and pharyngeal erythema.

Upper respiratory infection Possible complications: -Bacterial sinusitis -Lower resp. tract disease -Asthma exacerbation -Acute otitis media

A high VQ ratio is when __________ exceeds __________.

Ventilation exceeds perfusion. (not enough blood flow).

46 y/o male, PMH remarkable for HIV, presents to the clinic with gradual dyspnea on exertion, fever, and nonproductive cough. His O2 levels drop upon ambulation. You perform a CXR which reveals diffuse, bilateral interstitial or alveolar infiltrates. Labs show inc. LDH, inc. 1-3-beta-D-glucan, dec. PaO2, and inc. A-a gradient. What is your next step in determining the diagnosis?

You suspect Pneumocytosis in this patient. Immunofluorescent antibody staining or PCR of induced sputum or BAL Histopathology staining (Giesma, Silver) from sputum, BAL, or tissue bx. Treatment: Trimethoprim/Sulfamethoxazole Sulfa allergy: TMP-dapsone, primaquine-clindamycin, atovaquone, pentamidine If PaO2 <70mmHg & A-a gradient > 35mmHg: Gluticocorticoids

Mr. X is a 29 year old male who presents with acute onset of shortness of breath while training for his upcoming triathlon. Patient states he felt a "popping" sensation while running and soon after that experienced significant dyspnea on exertion. You note that he is a tall, thin male in great physical shape. CXR reveals a moderate left sided pneumothorax. Which of the following is the most common cause of primary spontaneous pneumothorax? a) Rupture of small blebs b) Emphysema and chronic bronchitis c) Pulmonary neoplasms d) Marfan's Syndrome e) TB

a) Rupture of small blebs

Complications of (microbe type) ________________ pneumonia include pleural effusion, empyema, pneumatoceles, necrotizing pneumonia, and lung abscesses.

bacterial

Mucopurulent sputum production is most frequently associated with _____________ pneumonia, while scant or watery sputum production is more suggestive of an atypical pathogen.

bacterial

Pneumonia has three patterns classically seen on __________: lobar pneumonia, bronchopneumonia, and interstitial pneumonia.

chest x-ray

The CURB-65 score mnemonic, used patients with community-acquired pneumonia, consists of _________________.

confusion, uremia, elevated respiratory rate, low blood pressure, and age greater than 65 years.

The most common opportunistic infection of the central nervous system in patients with AIDS is _____________ meningitis.

cryptococcal

In pleural effusions, tactile fremitus is (increased/decreased) _______________

decreased

In a patient with a recurrent malignant pleural effusion, which of the following is the most effective therapy in managing the patient symptoms? a) Thoracentesis b) Aggressive diuresis with lasix c) Radical pleurectomy d) Chemoradiation therapy e) Insertion of an indwelling pleural catheter

e) Insertion of an indwelling pleural catheter

A(n) ___________ pleural effusion has increased protein content and is often due to malignancy, pneumonia, collagen vascular disease, or trauma.

exudative

The influenza virus surface protein ________________ promotes its entry into cells.

hemagglutinin

Pulmonary hypertension is a disease that results in medial smooth muscle ___________, and intimal fibrosis of pulmonary arteries.

hypertrophy

Due to its diffuse patchy inflammation localized to interstitial areas at alveolar walls, atypical pneumonia is also known as _____________________.

interstitial pneumonia

Community-acquired pneumonia is defined as an acute infection of the pulmonary _____________ in a patient who has acquired the infection in the community.

parenchyma

The presence of ________________ on plain chest radiograph is considered the gold standard for diagnosing pneumonia when clinical and microbiologic features are supportive.

patchy infiltrate

The classic presentation of community-acquired pneumonia includes fever, productive cough (with/without) ________ purulent sputum, dyspnea, and pleuritic chest pain.

with


Kaugnay na mga set ng pag-aaral

Starting a Small Business Vocabulary

View Set

B101011 exam 1: Chapter 13, 14, and 15

View Set

Career Planning: The Job Search Process

View Set

Quiz 7: Increment and Decrement Operators

View Set

EXAM 1 Fin 2303-300 PART 1-Study

View Set

B-29 / Supervision of Battery Systems

View Set

Mizzou American Government Horner exam 4

View Set

Advanced Operating Systems Miderm

View Set